إعـــــــلان

تقليص
لا يوجد إعلان حتى الآن.

بشرى بمناسبه العيد لكل ممتحن

تقليص
X
 
  • تصفية - فلترة
  • الوقت
  • عرض
إلغاء تحديد الكل
مشاركات جديدة

  • بشرى بمناسبه العيد لكل ممتحن

    مين نازل امتحان في شوال او في الايام الجايه عندي مجموعه نمازج للي حيمتحنو من جده ام الرخى والشده وبالتوفيق للجميع

  • #2
    انا نازلة امتحان يمكن الاسبوع الجاي او اللي بعده ان شاء الله ومحتاجة لنماذج جديدة اذا ما عليكي أمر

    تعليق


    • #3
      والله تسوين فينا خير ..
      انا اختباري في كم اسبوع ..
      لو تقدري ترسليها ليا على البريد : dj.don@hotmail.com
      Emad : http://about.me/emad_abbas #
      Daily Deals Group : about.me/sfaqat1 #

      تعليق


      • #4
        1. اسئلة اخصائي يوم 10 -7-2012
        1__which the following with acetyl COA give acetic acid
        A_glucose
        B_lactate
        C_fatty acid
        D_oxalate
        2__Trichomonas vaginal صورة ل
        3__العضو الذي يعتبر استخدامه لfatty acid كطاقه يشكل خطوره هل
        A_ brain
        B_heart muscle
        C_Skeletal muscle
        D_liver
        انا اخترت الbrain
        4_organ use the glucose as flue
        A_brain
        b_pencrease
        c_kidney
        d_liver
        5__not ferment glucose
        A_salmonella
        B_shigella
        C_E.coli
        وفي اختيار رابع انا اخترت الرابع
        6__gram +ve in cluster
        s. aerous
        7__antibiotic against mRNA
        8__MCV 104 fl , vitaminB12 and folate abnormal
        A_Iron deficiency anemia
        b_liver cirrhosis
        c_auto haemolytic anemia
        d_vitamin E deficiency
        e_sideroblastic anemia
        9_Heinz bodies see in
        A_Reticulocyte smear
        وفي خيارات تانيه
        10_جدول في CBC and chemistry مطلوب AlG ratio
        11_sodium folurade for
        A_increase glycolysis
        b_decrease glycolysis
        c-inhabit glycolysis
        12_cryptopreciptate use for
        deficiency factor
        13_سؤال في بنك الدم قال مريض معاه lung injury ايش راح نستخدم معاه ...كل الخيارات خطأ ماعدا وحده صح
        A_unite contamination with bacteria
        b_irradiation of blood unite
        c_anti leukocyte for antibodies donor and antibodies pt
        وفي خيار رابع انا اخترت الثالث وطلع صح
        14_fresh frozen plasma all true except
        storage at 22 _26 C
        15_blood o group can donor to
        1_A
        2_b
        3_O positive
        4_AB
        الخيار الثالث
        16_female 65 years came to lab ANA positive ايش عندها
        1_rehamatic fever
        2_SLE
        وخيارات ثانيه
        17_platelate storage for
        5 days
        بكمل باقي الاسئله بعدين وبالتوفيق للجميع

        1. 18_term use when larva infiltration of tissue
        19_صوره لplate of macconkey ,وقال انو indol +ve and fermentation of lactose
        E.coli
        20_non motile for diagnosis we use stool
        1_enteric fever
        2_cholera
        3_shiglloesis
        انا اخترت الثالث
        21_principle of flamfotometer
        22_خلال كم ساعه malaria falciprum invasion of RBC
        1_12 hours
        2_24 hours
        3_48 hours
        4_72 hours
        23_virus ........principle of diagnosis
        24_lgA stimulate by
        25_وظيفه الADH .......water regulation
        26_neonatal meningitis from E.coli the best sample take from
        1_csf from Mather
        2_ csf from neonatal
        3_vaginal swap from mather
        4_nose swap from neonatal
        27_ adult tuberculosis of mycobacterium diagnosis by
        1_acid fast bacilli
        2_sputum culture in j m
        28_the cell of tuberculosis is
        1_fast acid bacilli
        29_water use in lab
        1_tap 1
        2_tap 2
        3_tap 3
        4_tap water هذا الخيار الصح
        30_high PCO2 cause
        1_respiratory acidosis
        2_respiratory alklosis
        3_metabolic acidosis
        4_metabolic alkalosis
        31_اللي يسبب الmetabolic alkalosis
        1_diarrhea
        2_sever vomiting
        3_strong exerciser
        4_hyper vent ration
        32_salmonilla in macconkey agar
        1_color less
        2_color less with black center
        3_red
        4_yellow _orange

        1. شكرا و هذا امتحاني في يوم الأربعاء 16/5/2012
        1. شكرا إليكم امتحاني:
        السلام عليكم و رحمة الله و بركاته هذا امتحاني في يوم الأربعاء 16/5/2012 و سامحوني على التأخير :
        بسم الله الرحمن الرحيم و سبحان الله و بحمده سبحان الله العظيم :
        الحمد لله نجحت ب 73% صحيح قليل بس الحمد لله
        *ملحوظة الإجابات قد تكون خطأ و التي أشك فيها أو التي يراد التأكد منها سأضع أمامها ? و لكن أرجو التأكد من الباقي. طبعا الأسئلة غير مرتبة هكذا لكني رتبتها ليتضح تكرار الأسئلة بصيغ أخرى و شكرا .
        1) The unit of measuring PO2 in blood is :

        a) mg/dl

        b) gm/dl

        c) ml/min

        d) mmHg

        e) mmol/l ?



        2)Blood gp. O contain Ab :

        a) Anti-A,B

        b) Anti-A only

        c) Anti-B only

        d) All of the above

        e) None of the above


        3)ABO antigens are :

        a) A,B,H

        b) A,B,O

        c) A,B,C

        d) A,B

        e) B,H

        4)Universal donor whose thus agglutinate all other except of its own due to it contain IgM is :
        a) O+?
        b) O-
        c) AB
        d) A+

        5) Ig consists of four polypeptide chain :
        a) IgM (pentamerخماسية الجزيئات و ليس سلاسل الببتيد )?
        b) IgG (monomer contain 4 polypeptide chainأظن هذه الإجابة الصحيحة لأنه: )
        c) IgA
        d) IgE

        6) Which is/are DAT :
        a) In vivo Ab in red cells surface
        b) In vitro Ab in red cells surface
        c) Incomplete Ab on red cells surface of donor
        d) All of the above

        7) Which is/are from agglutination reactions:
        a) DAT
        b) Detect Ab in red cell surface
        c) Latex
        d) Rf
        e) All of the above

        8) D r that permanently deferred if gives a positive history ofno
        a) HCV Ag
        b) HB core
        c) HIV
        d) All of the above

        9) To test Ab on red cell surface use :
        a) DAT
        b) IDAT
        c) …

        10) Pre-transfusion test for Ab is :
        a) IDAT
        b) DAT
        c) …

        11) Which of the following is/are from innate immunity :
        a) Salivary secretions
        b) GI normal flora
        c) Breast milk
        d) All of the above

        12) Humoral adaptive immunity component is :
        a) T-lymphocyte
        b) B-lymphocyte
        c) Ab ?

        13) Ab normally presented on red cell membrane is :
        a) Memory cell
        b) …

        14) Immunoglobin (Ig) produced by :
        a) Plasma cell
        b) Mast cell
        c) Fat cell

        15) B-lymphocyte produce :
        a) Ab
        b) Cytokines
        c) Sugar

        16) Macrophage is :
        a) Contain large dark blue granules
        b) Make phagocytosis to Ag
        c) Produce Ab
        d) …

        17) Largest WBC's in blood is :
        a) Lymphocyte
        b) Neutrophil
        c) Monocyte
        d) Basophil
        e) Eosinophil

        18) Neutropenia disease is :
        a) CML (chronic myeloid leukemia)
        b) …

        19) Thrombocytopenia disease is :
        a) Cytotoxic drugs
        b) Aplastic anemia
        c) Radiation
        d) Hemorrhage

        20) Philadelphia chromosome is diagnostic for :
        a) Acute lymphoblastic leukemia
        b) Acute myeloid leukemia
        c) Chronic myeloid leukemia
        d) …

        21) Monospot test used for :
        a) Infectious Mononucleosis
        b) Infectious lymphocytosis
        c) Acute lymphoblastic leukemia
        d) Acute myeloid leukemia

        22) When analysis anemia of normocytic normochromic to exclude hemolytic we do :
        a) CBC
        b) Ferritin
        c) Transferrin
        d) Soluble transferrin … sT..لا أذكر باقي اسم التحليل و اختصاره ?

        23) Decrease neutrophil , lymphocyte , … :
        a) Pure red cell aplasia
        (notes : pure red cell aplasia : RBC's and PCV only are low but WBC's normal)
        b) Aplastic anemia
        c) …
        d) All of the above

        24) Decrease neutrophil , lymphocyte , platelets , … :
        a) Pancytopenia
        b) …

        25) Bence-Jones protein presented in :
        a) Multiple myeloma
        b) Sideroblastic anemia
        c) …

        26) Retics normal range in adult is :
        a) 1-5
        b) 0.1-0.2
        c) 0.2-5
        d) 0.2-2 ?

        27) ESR anticoagulant is :
        a) EDTA
        b) Sod-citrate
        c) Sod-citrate + heparin
        d) Li-heparin

        28) Plain tube should be :
        a) Clotted
        b) Gently mixed
        c) Hemolysis

        29) Westergen technique is used for :
        a) ESR
        b) …
        30) Gout disease caused by increase of :
        a) Uric acid
        b) Glucose
        c) …

        31) Glycosylated Hb used to test glucose in last :
        a) 1-2 weeks
        b) 6-12 weeks
        c) 12-16 weeks ? (notes : 3months=90days=12.9weeks)
        d) 16-18 weeks

        32) Flame photometer to measure :
        a) Na
        b) Urea
        c) …

        33) Atomic absorption photometer lamp is :
        a) Hollow cathode …
        b) Halogen
        c) Tungsten
        d) …

        34) Fluorophotometer lamp is :
        a) Halogen ?
        b) Hollow cathode
        c) Tungsten
        d) Laser
        e) …

        35) Nephlometer and turbidometer to detect light :
        a) Emitted by small granules
        b) Absorbed by large granules
        c) Reflected by small granules ?
        d) Reflected by large molecules
        e) …

        36) To measure turbidity detect light:
        a) Emitted by small granules
        b) Absorbed by large granules
        c) Reflected by small granules ?
        d) …

        37) Electrophoresis depends on :
        a) Charge
        b) Size
        c) …

        38) For best analysis to Na , K :
        a) Chromatography
        b) Iron selective electrode (ISE)
        c) Protein chromatography

        39) Normal range for glucose after taking carbohydrate meal is :
        a) 70-110
        b) 100-120
        c) 120-150 ?
        d) …

        40) In 2 fold serial dilution the dilute in tube number 5 is:
        a) 1/24
        b) 1/32
        c) 1/64
        d) 1/256
        (ملحوظة في محاضرة أبو راشد رقم 2 من موقع مختبرات العرب شرحه لا ينطبق على الاختيارات المتاحة فأبو راشد يقول :
        (tube 1=undiluted sample , 2=1/2 , 3=1/4 , 4=1/8 , 5=1/16

        41) When put 20µ serum to 480µ diluent dilution will be:
        a) 1/25?
        b) 1/50
        c) …

        42) Hormone increase normally during pregnancy and lactation:
        a) Prolactin (PRL)
        b) TSH
        c) PTH
        d) Growth hormone (GH)

        43) Which is/are from steroid hormones :
        a) Glucocorticoid
        b) Insulin
        c) Glucagon
        d) All of the above

        44) Which the following catheters collect urine specimen in prolonged period:
        a) Foley?
        b) Italian
        c) Soft
        d) Hard
        e) …

        45) G+ve bacteria cell membrane contain :
        a) Lipopolysaccharide
        b) Phospholipid +glucose
        c) Phospholipid +cellulose
        d) Toxic Peptidoglycan…?
        e) …

        46) General stain most used in bacteriology to differentiate microorganism :
        a) Gimesa
        b) Ziel-Neelsen (ZN)
        c) Gram
        d) Leishman

        47) Which is not from ZN-stain contents :
        a) Carbol fuchsin
        b) Crystal violet
        c) Malachite green
        d) Methylene blue

        48) G-ve bacteria are stained :
        a) Purple
        b) Pale to deep red
        c) Blue
        d) Orange

        49) Which isn't streptococci :
        a) S.pneumoniae
        b) S,pyogenes
        c) S.aures
        d) S.agalactiae

        50) G-ve rod capsulated is :
        a) Proteus
        b) Klebsiela
        c) Salmonella
        d) Yersinia
        e) Shigella

        51) Small G-ve bacillus anaerobic is/are :
        a) Lactobacillus
        (I think it's lactobacillus only because it's vaginal normal flora anaerobic but Acinetobacter is skin normal flora aerobic & Bacillus is aerobic)?
        b) Acintobacter
        c) Bacillus
        d) a&b

        52) Which is realy TB on Lowenstain-Jensen(L-J) medium :
        a) Dry , yellow
        b) Buff , dry
        c) Yellow , moist…
        d) None of the above
        (notes : dry , cream)

        53) Which is/are enterococcus :
        a) Gp. D
        b) Enterococcus faecalis
        c) Faecum
        d) All of the above

        54) G-ve cocci cause abscess , endocarditis , … :
        a) Diplococci (G-ve cocci as Neisseria الوحيدة اللي)
        b) Streptococci
        c) Staphylococci
        d) …

        55) G-ve bacilli cause wound , UTI , blood stream infections :
        a) Enterobacteriacae
        b) Streptococci
        c) Staphylococcid
        d) … (لا أذكر هذا الخيار)

        56) Eye swab cultured on :
        a) Blood only
        b) MacConkey only
        c) Blood + Chocolate
        d) …

        57) Chocolate agar used for :
        a) Haemophilus
        b) …(لا أذكر باقي الخيارات)

        58) G-ve bacilli oxidase +ve non-spore :
        a) Pseudomonas
        b) …(لا أذكر باقي الخيارات)

        59) Herpes virus type I cause :
        a) Herpes?
        b) …(لا أذكر باقي الخيارات)

        60) Gas gangrene caused by :
        a) Cl.perfringers
        b) Cl.tetani
        c) Cl.botilinum

        61) We use in catalase :a) H2O2b) …(لا أذكر باقي الخيارات) 62) Parasite transmitted by animal :
        a) Trypanosoma(by TseTse fly) ? I
        (لا أدري أهذه هي الحيوان أم الخيار الذي لا أذكره)
        b) Entamoeba histolytica
        c) Gardia lamblia
        d) …(لا أذكر باقي الخيارات)

        63) Entrobius vermicularis infective stage is :
        a) Larvae
        b) Cercaria
        c) Egg

        64) S.mansoni infective stage is :
        a) Larvae
        b) Cercaria
        c) Egg

        65) Which is from tape worms :
        a) Anclystoma
        b) Wucherreria
        c) T.saginata

        66) Lymphatic filarial is :
        a) Trypanosoma
        b) Wucherreria bancrofti
        c) …

        67) Leishmania is transmitted by :
        a) TseTse fly
        b) Anopheles
        c) Phlobotomus Sand Fly

        68) Toxoplasmosis is/are :
        a) Cause fetal cerebral infection
        b) Detected by serology
        c) Infect cats
        d) All of the above
        11-03-2011, 01:57 AM #91
        هدى عسيري
        عضو جديد
        تاريخ التسجيل
        Feb 2011
        الدولة
        المدينه المنورة
        المشاركات
        10
        الحمدلله اختبرت يوم الاربعاء ونجحت
        هادي الاسئله اللي افتكرتها وكتبتها اول ماخرجت طبعا صيغه السوال مو نفسها بس الاجابة وحده عشان كدا لازم تركزو في السوال
        Fixed tissue macrophages are
        دا السوال جا انو كل الخيارات التاليه خطا عن
        المونو سيت ماعدا اللي هي وظيفه الابتلاع
        a. Basophilesهنا جا سؤال عنها انها اقل عددا
        b. Monocyts
        c. Lymphocytes
        d. Neutrophils
        e. Eosinophils.
        A guideline reference range for MCHC in health patient
        a. 80-98 fl.
        b. 27-32 pg
        c. 31.5-36 g/dl.
        d. 15.18 mg/dl.
        e. 13.5-18 g/dl.
        The best anticoagulant used for determination of erythrocytes sedimentation rate is:
        • Na citrate
        • EDTA
        • Heparin
        • None of the above
        In DIC all the following is increased except
        a. PT
        b. APTT
        c. TT
        d. Platelets
        None of the above
        HAV transmission
        a. Blood
        b. Body fluid
        c. Sexual contact
        d. Fecal oral
        Enterococci are typically arranged in pairs and can not be or difficult differentiated from
        a. Streptococci
        b. Staphylococci
        c. Spirochete
        d. Meningococci
        Patient comes to the lab. After take meal since two hours the normal level of his glucose is
        a. 100 – 150 mg/dl.
        b. 70 – 110 mg/dl.
        c. 50 – 90 mg/dl.
        100 – 150 gm/
        Which is most abundant (prevalence)?
        a. Plasmodium falciparum.جا سوال عنوبلازموديوم ينتج ماء اسود انا اخترت دا
        b. P.ovali.
        c. P.malari.
        P.vivax

        Blood grouping antibodies are
        a. IgG
        b. IgM جا سؤالين عنو ايت واحد حق التحليل او الاصابه الحديثه برايمري اميونو
        c. IgE
        d. IgD
        e. IgA
        A good complement fixing antibodies
        a. IgG
        b. IgM
        c. IgD


        Antibodies associated in Rh
        a. IgGجا سؤال عنو التحليل او الاصابه القديمه اللي هو السكندري اميونو
        b. IgM
        c. IgE
        d. IgD
        e. IgA
        Congenital Parasites infected human
        a. Entamoeba histolytica
        b. Giardia lamblia
        c. Trypanosoma sp.
        d. Plasmodium malaria
        e. Toxoplasma gondii السوال جا انو هذا الكائن يشخص في السرولوجي
        The first aid you do in swallowing of an acid or alkali chemical or poisons.
        a. Wash immediately in running water
        b. Seek medical attention
        c. Trans to other place
        d. Call the physician
        e. Immediately rinse the mouth well with water.
        A case conjunctivitis has been suspected to be trachoma . wich of the following organisms is MOST likely to be the causative agent :جا زي ماهو بالنص بالزببببط
        • Ricketsia species
        • Boreerlia vincenti
        • Clamydia species
        • Treponema pallidum
        • Borrelia duttoni
        The biochemical test differentiate S.pneumonia from other streptococci
        a. Catalase
        b. Coagulase
        c. Citrate
        d. Bile solubility
        Bacillary dysentery caused by
        a. Shigella جا لها سوالين دا السوال والتاني ان كل الخيارات صحيحه للشجيلا ماعدا انها تخمر الالكتوز(طبعا هي ماتخمر الاكتوز)
        b. Cambylobacter
        c. Chylamidia
        d. Salmonella
        A carbon dioxide enriched atmosphere is required for the culture of : جا دا السوال بالنص بس ماكان في All the bove
        • Bacillus anthracisانا اخترت دي ومدري صح ولالا
        • Heamophilus influnzae
        • Brodetella pertussis
        • Pseudomonas aeruginosa
        • Yersinia pestis
        • All the bove
        Hormones from cortex
        a. Thyroxin
        b. Oxytocin
        c. Prolactin
        d. Cortesol
        Philadelphia chromosome is a specific chromosomal abnormality that is associated with.
        a. ALL
        b. AML
        c. CML
        d. None.
        Cells not contain nucleus.
        a. Erythrocyte
        b. Leukocyte
        c. Platelets
        d. blast cell
        T. cell maturation in
        a. Bon marrow
        b. Thymus
        c. Spleen
        d. Liver
        Which of the following organism oxidase positive
        a. Psedomonas only
        b. Nessieria only
        c. E.coli
        d. Psedomonas & nessieria
        Which of the following is true?
        a. RH-ve person safely donate for person RH+ve person
        b. RH+ve person safely donate for person RH-ve person
        c. A blood group person donate for B group person
        d. All of the above
        In Pregnancy test we measure
        a. LDH
        b. Estrogen
        c. α HCG
        d. βHCG
        Normal range of uric acid?
        a. 2-5 mg/dl.
        b. 3-7
        c. 5-10
        d. 70 -110 mg/dl
        Cause Kala- azar:
        a. leishmania tropica
        b. leishmania braziliense
        c. leishmania donovani
        d. leishmania Mexicana

        All of the following test use in myocardial except
        a. Creatinin
        b. Creatinin clearance
        c. sGPT
        d. sGOT
        Adrenal gland upside of?
        a. Liver
        b. Kidney
        c. Brain
        d. Heart
        Eosinphil raised in the following except?
        a. Allergic
        b. Parasitic disease
        c. Hodgkin disease
        d. Bacterial disease
        Test for Intrinsic pathway:
        a. Bleeding time
        b. Thrombin time
        c. Prothrombin time
        d. Partial thromboplastin time PTT
        The contamination of lens of microscope which cause eye infection?
        a. B. anthrax
        b. S.pyogenes
        c. Staph.aureus
        d. Clostredium tetani
        Pathogens can enter the body through?
        a. Dry skin
        b. Wet skin
        c. Unbroken skin
        d. Scratch skin
        More abundant leukocyteجاس وال مرره طويل وماافتكرو بس كان دا الجواب والمهم كان من ضمن السوال انها توجد بكميه كبيره
        a. Basophile
        b. Monocyte
        c. Lymphocyte
        d. Neutrophils
        e. Eosinoph
        One test in the following can diagnose anemia :
        a. CBC
        b. PT
        c. PTT
        d. Platelets
        e. Leukocyte
        Substance can not trans and absorbed without a factor
        a. Iron
        b. Vitamin B12
        c. Folic acid
        d. Vitamin A
        chronic carrier state may occur in
        a. HBV
        b. AIDS
        c. E.coli
        d. Mumps
        e. Rubella
        The total leukocytes count less than 4,000/cmm is found in
        a. Typhoid infection
        b. Leukopenia
        a. S.Pyogenes
        c. A and B


        Lenses near the slide in light microscope:
        a. Objective
        b. Stage
        c. Eye lenses
        Storage temperature of platelets
        a. 37 C
        b. 35 – 40
        c. 22 – 27
        d. -4
        Hb range in adult:-
        a. 13.5 - 17.5 gm/dl.
        b. 12 -16 gm/dl
        c. 13.5 - 17.5 mg/dl.
        d. 11 – 15 gm/dl.
        The following parasite will not take the gimsa stain:
        a. leishmanai.
        b. Trypanosoma
        c. Schistosoma
        d. Malaria
        e. Filaria
        The antibody of blood group is AB:
        a. Anti A and B
        b. Anti A and C
        c. Anti A and O
        d. Non All of the above
        All of the following bacteria cell components except
        a. Cell wall
        b. Cell membrane
        c. Nucleus
        d. Cytoplasm
        e. Pili
        Heprenized syringe use to collect __________ sample
        a. Synovial
        b. CSF
        c. Venous
        d. Arterial
        Streptococci are
        a. α-hemolytic
        b. β-hemolytic
        c. γ-hemolytic
        d. ALL
        Serum is plasma without
        a. Lipid
        b. Cholesterol
        c. Fibrinogen
        d. Fibrin
        CPDA anticoagulant to store blood for :
        a. 45
        b. 21
        c. 10
        d. 35
        Zile-Neelsen stain for stainingماجاني دا الكائن جا واحد تاني اسمو غريب واخترتو انتو دورو ايت كائن يصبغ بهادي الصبغه
        a. Mycobacteria
        b. E.coli
        c. Corynebacterium diphteria
        d. Salmonella
        During reaction of 2 chemical substances , the color produced asses by :
        a. Spectrophotometer
        b. Flame photometer
        c. ELISA
        Flourometry use (S) _________ as source of light .
        • Deuterium lamp
        • Mercury-arc lamps
        • Halogen lamp
        • Hallow high energy cathode lamp ( HCL )
        • Tungsten lamp
        Thromobocytosis is a common finding in which of the following diseases :
        • Von willebrand disease
        • Immune thromobocytosis purpura
        • Iron deficiency anemia
        • Sickle cell anemia )
        • Glanzmann's thrombasthenia
        All of the following are classified as streptococci EXCEPT :
        • S. Agalactiae
        • S. pyogenes
        • S. mutants
        • S. Aureus
        Normal value of Lymphocyte
        a. 55%
        b. 25%
        c. 70%
        Infective stage of hook worms
        a. Larva
        b. Egg
        c. Cyst
        d. Metacercaria
        Coombs test used for
        a. Auto antibodies on surfase RBCs
        b. Antigene RBCs
        c. Antibodies in serum
        d. Antigene on Cells
        A heamocytometer cannot be used for the counting which of the following :
        • Red blood cells
        • Sperm
        • Total white blood cells
        • Lymphocytes
        • Platelets
        -جا سؤال عن جهاز الكروماتوجرافي يستخدم لماذا انا جاوبت اللون
        الطور المعدي للدوده الدبوسيه ال EGG
        Test differentiate staphylococci from streptococci
        a. Catalase
        b. Coagulase
        c. Citrate
        d. Bile solubility

        بكتريا الهليوبكترمميزاتها؟؟+ve coccobacilli
        كل الخيارات تستخدم في الفرن الجاف ماعدا؟gloves
        بكترياc.botulism تسبب botulism
        جرام بوزتيف كابسول؟؟ كان الخيارات ستاف وستربتو و meningococci
        BHS increase??sickle cell anemia
        1. جديد نموذج أسأله الهيئه اخصائي مختبر بتاريخ 14/6/2012 ارجو من الله التوفيق للجميع

        Harada Moro technic for one of
        1- Stringloid
        2- Hook worm
        3- Ascaris
        ________________________________
        Hashimoto Disease Ab سؤال عن
        ______________________________________
        Pseudomonas Aerogenosa is
        1-lophotrichus
        2- monotrichus
        3-Amphotrichus
        4- Atrichus
        __________________________________________
        B-Lactmase Bacteria is resistant to
        1 –penicillin
        2 – tetracycline
        3 – cotrimoxazol
        __سؤال عن تركيب الهيموجلوبين
        Hb A or HbA2 or HbF
        المهم واحد مهم
        يعني كم بيتا وكم الفا او جاما
        __________________________________
        In PCR copy of DNA
        يعني كيف يحصل نسخ للدي ان اه
        وجايبين اختيارات منها
        Amplification of complement DNA
        ______________________________________
        Pence Jons Protein found in
        Multiple myloma

        ______________________________________
        سؤال يقول ماهو البروتين اللذي يحمل عليه TSH
        وعاطي اختيارات
        1- albumin
        2- thyroglobin
        ___________________________
        RPR Test for
        Syphilis
        _________________________________________________
        How we different between group A & Group B streptococci
        1-Bactracin
        2- optichin
        3- neomycin
        1. ______________________Eosinophilia in cases of
        Parasitic And allergic
        _____________________________________________

        Secretary Ab
        1- IgA

        _______________________________________________
        All are true about Salmnella Exept
        1- H2S Producton
        2- Non-lactose fermenter
        3- Enteric fever
        4- Lactose fermenter

        ______________________________________
        سؤال عن ًWrite stain
        __________________________________________________ _
        Pad cholesterol & good Cholesterol
        LDL (Pad) HDL (Good)
        __________________________________________
        Catalase to different between
        1 – staph & strept
        __________________________________________________ _______
        Bacteria secret cAMP factor
        1 – Strept Agalactica

        __________________________________________________ ____________
        Test specialist for enterococci
        Litmus milk decolonization

        __________________________________________________ ____________
        Septicimia cane caused by
        1 – brucella
        2- shigella
        3- EHEC
        4- ETEC
        __________________________________________________ _________
        Parasite in crescent shape
        Plasmodium falciparum

        __________________________________________________ __________

        صوره طبق مزروع عليه بكتيريا نوع الطبق نيترنت اجار والسؤال
        This bacteria isolated from patient with UTI this bacteria make swarming
        Proteus
        رد مع اقتباس
        1. Man 45 y with abd.pain doctor request stool c/s the result
        Gram + rod collect in chines litters this bacteria is
        Corynobacteria diphtheria

        __________________________________________________ _
        Man age 25y take swab from wound and inculate in blood agar
        Result G + rod beta heamlytic spor forming
        Clostridium perfringens

        __________________________________________________ __
        Infant age 6 month with differential leukocyte as following
        Neutrophils 40%
        Lymphocytes 50%
        Monocytes 6%
        Eosinophiles 4%
        Basophils 0%
        This is
        1- bacterial infection
        2- viral infecton
        3- entric fever
        4- normal case

        __________________________________________________ _____________
        رجل عمره 42 سنه يسوق سياره وعمل حادث ونقل الى الطوارئ خلال عمل الفحوصات وجد عنده التالي
        1- RBS 95 mg/dl
        2- AST 43 U/l
        3- CK high increase 3 times increase
        Your diagnosis for this case
        1 –liver injury
        2 – liver abscess
        3 – muscle injury
        4 – myocardial infraction

        __________________________________________________ ________________

        تعليق


        • #5
          وكم يوم ان شاءالله وانزل الباقي

          تعليق


          • #6
            يااخ عماد انت دبلوم وهذه اسئله اخصائي

            تعليق


            • #7
              الله يجزاكي خير انا عندي امتحان يوم السبت بعد يومين اذا في مجال اي اساله او اي مساعده ياجماعه انا اخصائي مختبر الرجاء ارسال اي شيئ على aali988at hotmail.com

              تعليق


              • #8
                تسلمي اخت داليا في موازين حسناتك يارب وموفقة بالامتحان

                تعليق


                • #9
                  اللهم امين ربنا يوفقنا ويوفق الجميع

                  تعليق


                  • #10
                    1. الحمد لله رب العالمين بفضل الله عز وجل اجتزت الامتحان اليوم بمجموع 57% بمعجزه من الله عز وجل
                    اولا انا اسف على التأخر والله لسه راجع البيت حالا
                    ثانيا الاسئله اللى جت من مذكرة الاخت قطرة مطر لا تتجاوز ا 5 سؤال وفيه منهم بطريقه غير مباشره وباقى الاسئله والله ربنا لطف بيا لانى مكنتش اعرف عنها حاجه
                    لكن الله عز وجل لطف بيا جدا
                    ومن الاسئله اللى اذكرها

                    10. Leukocytes involved in anaphylactic hypersensitivity and inflammatory reaction
                    a. Basophiles
                    b. Monocyts
                    c. Lymphocytes
                    d. Neutrophils
                    e. Eosinophils
                    بس فيها كلام زياده
                    Gram positive rods give gray colony color
                    a. Bacillus anthraces
                    b. E.coli
                    10
                    c. Shigella
                    d. Mycobacterium
                    e. Non

                    Anticoagulant used in ESR test
                    a. Sodium citrate
                    b. Ammonium oxalate
                    c. SPS
                    d. EDITA
                    e. Heparin
                    بس طبعا ما جابها بالطريقه دى ذكر اسم اللى اس ار كلها وقال na citrate
                    Congenital Parasites infected human
                    a. Entamoeba histolytica
                    b. Giardia lamblia
                    c. Trypanosoma sp.
                    d. Plasmodium malaria
                    e. Toxoplasma gondii
                    هتذكر باقى الاسئله ان شاء الله واكتبها
                    وربنا معاكو يا رب بس الاسئله محتاجه تركيز جامد وقرأه جيده

                    وجانى سؤال عن البلانتديوم كولاى كيفية التشخيص
                    A Carbon dioxide enriched atmosphere is required for the growth of
                    بس مانى فاكر الخيارات
                    وجتنى اسئله عن الجمره الخبيثه اكثر من سؤال
                    وجانى سؤال عن ال N.gonorrhoea كيف بتشخص عن طريق مسحه من اين
                    The process of pathogenic infection to human
                    a. Adherences – Penetration –Multiplication
                    b. Penetration –Adherences- Multiplication
                    c. Multiplication – Adherences- Penetration
                    وجانى سؤال عن ال double zone
                    The antigene in ABO system
                    a. A.B.C
                    b. A.B.O
                    c. A.B.H
                    d. A.B.D
                    The foundmental of erythrocyte production is:
                    a. Erythropiotein
                    b. Thyroxin
                    c. Growth hormones
                    d. ADH
                    وسؤال عن الهرمون اللى بينتج من الفص الامامى للغده النخاميه
                    وسؤال عن الفرست ايد
                    وعن معادلة الحمض
                    والتركيز على الانيميا
                    واكرر ياريت نقرأ الاسئله كويس بتمعن
                    واسف على باقى الاسئله لانى اساسا مش فاكرها اصلا واسف على طريقة العرض
                    التعديل الأخير تم بواسطة ابو يزيد3212 ; 11-07-2012 الساعة 02:13 AM

                    تعليق


                    • #11
                      نموذج (1



                      1- Vitamin K antagonist :
                      a- warfarin
                      b- Heparin
                      c- Protein C
                      d- Antithrombin III
                      2- One of the intrinsic pathway
                      a- factor XI
                      b- factor XIII
                      c- factor I
                      d- factor VII
                      3- Para hemophilia is the deficiency of
                      a- factor VIII
                      b- factor IX
                      c- factor V
                      d- factor VII
                      4- Eosinophilia is seen in :
                      a- food sensitivity
                      b- Drug sensitivity
                      c- Atopic dermatitis
                      d- all of the above
                      5-Multiple myeloma is a neoplastic proliferation of:
                      a- lymphocytes
                      b- Granulocytes
                      c- Plasma cells
                      d-Monocytes
                      6- Test for intrinsic pathway:
                      a- bleeding time
                      b- Thrombin time
                      c- Prothrombin time
                      d- Partial thromboplastin time
                      7- Paul-Bunnel test is done to diagnose:
                      a- multiple myeloma
                      b- Hodgkin’s disease
                      c- Infectious mononucleosis
                      d- all of the above
                      8- increased platelet count is :
                      a- thrombocytopenia
                      b- thrombopoietin
                      c- thrombocytosis
                      d- all of the above
                      9- Decreased platelet count is:
                      a- thrombocytopenia
                      b- Thrombopoietin
                      c- Thrombocytosis
                      d- all of the above
                      10- All these are causes of thrombocytopenia except:
                      a- cytotoxic drugs
                      b- Aplastic anemia
                      c- Hemorrhage
                      d- Radiotherapy
                      11- Prothrombin time is done to test:
                      a- Intrinsic pathway only
                      b- Extrinsic pathway only
                      c- Extrinsic and common pathways
                      d- Intrinsic and common pathways
                      12- Normal bleeding time by Duke’s method:
                      a- 2-7 minutes
                      b- 2-7 seconds
                      c- 2-4 minutes
                      d- 2-4 seconds
                      13- Normal partial thromboplastin time (PPT) is :
                      a- 3-4 minutes
                      b- 30-45 seconds
                      c- 12-15 seconds
                      d- 12-15 minutes
                      14- Hemophilia A is the deficiency of :
                      a- factor V
                      b- factor VIII
                      c- factor IX
                      d- all of the above
                      15-the most common form of leukemia in children is:
                      a- acute lymphoblastic leukemia
                      b- Chronic lymphocytic leukemia
                      c- Acute myeloid leukemia
                      d- Chronic myeloid leukemia
                      16- Bence-Jones protein is present in cases of:
                      a- chronic myeloid myeloma
                      b-acute myeloid myeloma
                      c- Hodgkin’s lymphoma
                      d- multiple myeloma
                      17- Reed-Sternberg cells are found in cases of :
                      a- acute lymphoblastic leukemia
                      b- Non Hodgkin’s lymphoma
                      c- Hodgkin’s lymphoma
                      d- Multiple myeloma
                      18- Normal platelet count is :
                      a- 150,000 to 450,000/min3
                      b- 400,000 to 800,000/min3
                      c- 4,000 to 11,000 /min3
                      d- 50,000 to 100,000/min3
                      19- Antithrombin III inhibits:
                      a- factor Va
                      b- factor VIIIa
                      c- factor Xa
                      d- all of the above
                      20- Heparin potentiate the action of :
                      a- protein C
                      b- protein S
                      c- antithrombin III
                      d- warfarin
                      21- Factor II of blood clotting is:
                      a- Christmas factor
                      b- Fibrinogen
                      c- Prothrombin
                      d- Thromboplastin
                      22- One of fibrinogen group is :
                      a- II
                      b-V
                      c- VII
                      d- IX
                      23- Fibrinogen is converted to soluble fibrin by:
                      a- prothrombin
                      b- Thromboplastin
                      c- Thrombin
                      d- all of the above
                      24- Thrombopoitin control the formation of:
                      a- red blood cells
                      b-White blood cells
                      c- platelets
                      d- non of the above
                      25- Normal prothrombin time (PT) is:
                      a- 30-45 seconds
                      b- 30-45 minutes
                      c- 12-15 seconds
                      d- 12-15 minutes
                      26- Parasitic disease is associated with:
                      a- monocytosis
                      b- Lymphocytosis
                      c- Basophilia
                      d- Eosinophilia
                      27- Philadelphia chromosome is diagnostic for:
                      a- acute lymphoblastic leukemia
                      b- Acute myeloid leukemia
                      c- Chronic lymphocytic leukemia
                      d- chronic myeloid leukemia
                      28- Normal fibrinogen level:
                      a- 150-400 gm%
                      b- 150-400 mg%
                      c- 15-40 mg%
                      d- 15-40 gm%
                      29-infectious mononucleosis is caused by:
                      a- echo virus
                      b- coxsaki virus
                      c- Epstein Barr virus
                      d- Cytomegalo virus
                      30- Atypical lymphocytosis is seen in cases of:
                      a- Hodgkin’s lymphoma
                      b-Multiple myeloma
                      c- Infectious mononucleosis
                      d- Chronic lymphocytic leukemia
                      31-monospot test is done to diagnose:
                      a- Acute myeloid leukemia
                      b- Acute lymphoblastic leukemia
                      c- Infectious mononucleosis
                      d- Infectious lymphocytosis
                      32- The test which depend on blood platelets & capillary fragility is:
                      a- prothrombin time
                      b- Thrombin time
                      c- Bleeding time
                      d- Clotting time
                      33- Fibrin is broken to fibrin degradation products by the action of:
                      a- Prothrombin
                      b- Thrombin
                      c- Plasminogen
                      d- Plasmin
                      34- Acute myeloid leukemia is characterized by:
                      a- low neutrophil alkaline phosphatase
                      b-Myeloblast with Auer rods
                      c- Neutrophil with Pleger-Huet anomaly
                      d- all of the above
                      35- Plasminogen is converted to plasmin by :
                      a- Heparin
                      b-Histamine
                      c- Urokinase
                      d- Serotonin
                      36- Increase D-dimers and fibrin degradation products are seen in cases of:
                      a- Hemophilia A
                      b- Vitamin K deficiency
                      c- Diffuse intravascular coagulation
                      d- Von Willebrand disease
                      37- Activated protein C degrades:
                      a- factor IXa
                      b- Factor VIIIa
                      c- Factor Xa
                      d- Factor Xia
                      38- Heparin is found in
                      a- Neutrophil
                      b- Basophil
                      c- Acidophil
                      d- all of the above
                      39- In hemophilia A the test which will be prolonged is
                      a- PT
                      b- PTT
                      c- Bleeding time
                      d- all of the above

                      40- Bleeding due to overdose of heparin is managed by giving:
                      a- Vit K
                      b- Vit C
                      c- Vit A
                      d- Protamin sulphate
                      41- Streptokinase and staphylokinase convert:
                      a- Prothrombin to thrombin
                      b- Fibrinogen to fibrin
                      c- Soluble fibrin to insoluble fibrin
                      d- Plasminogen to plasmin
                      42- Test for platelet function:
                      a- Clot retraction
                      b- Platelet aggregation
                      c- Platelet adhesion
                      d- all of the above
                      43- Prolonged PT occurs in cases of deficiency of:
                      a- Factor III
                      b- Factor IV
                      c- FactorV
                      d- all of the above
                      44- normal thrombin time (TT):
                      a- 30-45 sec
                      b- 2-4 min
                      c- 3-9 min
                      d- 10-20 sec
                      45- cause of vitamin K deficiency:
                      a- Obstructive jaundice
                      b- Prolonged use of antibiotics
                      c- Inadequate intake
                      d- all of the above
                      46- Cause of Hyper- Coagulable state:
                      a- Aplastic anemia
                      b- Cytotxic drugs
                      c- Polcythemia

                      d- Radiotherapy
                      47-Physiological cause of neutrophilia:
                      a- New born
                      b- Radiotherapy
                      c- Cytotoxic drugs
                      d- Prolonged use of antibiotics
                      48- Leucocytosis characterized by the presence of immature cells and high
                      neutrophil alkaline phosphatase:
                      a- chronic myeloid leukemia
                      b- Acute myeloid leukemia
                      c- Leukaemid reaction
                      d- non of the above
                      49- Normal coagulation time (CT):
                      a- 3-9 min
                      b- 3-9 sec
                      c- 30-40 sec
                      d- 30-40 min
                      50- The test which measures the clotting time of citrated plasma accelerated
                      by the addition of a clotting factor activator (kaolin) , phospholipids and
                      calcium:
                      a- coagulation time
                      b- Prothrombin time
                      c- Partial thromboplastin time
                      d- Thrombin time
                      51- The test which measures the clotting time of citrated plasma to which
                      thromboplastin and calcium has been added:
                      a- thrombin time
                      b- Prothrombin time
                      c- Coagulation time
                      d- Partial thromboplastin time
                      52- The test which is widely used as a control and follow up test to control
                      anticoagulant treatment:
                      a- APTT
                      b- PTT

                      c- PT
                      d- TT
                      53- A disease characterized by progressive neoplastic proliferation of
                      immature white cell precursor:
                      a- acute leukemia
                      b- Chronic leukemia
                      c- Lymphoma
                      d-Multiple myeloma
                      54- The absolute lymphocyte count may be up to 300,000 or more between 70
                      and 90%of white cells in the blood film appear as small lymphocytes . THE
                      CASE IS:
                      a- Acute myeloid leukemia
                      b- Acute lymphoblastic leukemia
                      c- Chronic myeloid leukemia
                      d- Chronic lymphocytic leukemia
                      55- Variation in red cells size:
                      a- Poikilcytosis
                      b- Anisocytosis
                      c- Reticulocytosis
                      d- Leukocytosis
                      56- Dark red cells with no area of central pallor:
                      a- Stomatocyte
                      b- Sherocyte
                      c- Acathocyte
                      d- Schistocyte
                      57-Microcytic hypochromic anemia
                      a- hereditary spherocytosis
                      b- Sickle cell anemia
                      c- Iron deficiency anemia
                      d- Vit B12 deficiency anemia
                      58- Target cells are seen in cases of:
                      a- folic acid deficiency
                      b- Iron deficiency anemia
                      c- Vit B12 deficiency anemia

                      d- Thalassemia
                      59- Red cells with elongated area of central pallor:
                      a- spherocyte
                      b- Schistocyte
                      c- Stomatocyte
                      d- Elliptocutes
                      60- ……………….. Symmetric, short , sharp projection from the red cells and
                      seen in iron deficiency anemia:
                      a- echinocyte
                      b- Acanthocyte
                      c- Elliptocyte
                      d- Ovalocyte
                      61- ………………is a condition in which the absorption of vit B12 is greatly
                      impaired due to failure or marked reduction of intrinsic factor secretion:
                      a- fauvism
                      b- fanconi’s anemia
                      c-sickle cell anemia
                      d- thalassemia
                      62- Hyperchromic cells are seen in:
                      a- iron deficiency anemia
                      b- Thalassemia
                      c- Hereditary spherocytosis
                      d- Sickle cell anemia
                      63- A prolonged low rate of bleeding results in:
                      a- normochromic anemia
                      b- Hypochromic anemia
                      c- Hyperchromic anemia
                      d- non of the above
                      64- Schilling test is done in diagnosis of:
                      a- iron deficiency anemia
                      b- Pernicious anemia
                      c- Aplastic anemia
                      d- folic acid deficiency

                      65- Defective synthesis of either alpha or beta chains of normal hemoglobin
                      cause:
                      a- sickle cell anemia
                      b- Aplastic anemia
                      c- Pernicious anemia
                      d- Thalassemia
                      66- Neutrophils represent……………of circulating leukocyte:
                      a- 2-8%
                      b- 0-1%
                      c- 50-70 %
                      d- 2-4%
                      67- …………. are non nucleated, biconcave shaped cells:
                      a- platelet
                      b- Leukocyte
                      c- Erythrocyte
                      d- Macrophages
                      68- …………….. represent 50-70%of total leukocytes
                      a- lymphocytes
                      b- Neutrophils
                      c- Monocytes
                      d- Eosinophilis
                      69- ……………have a characteristic biffed nucleus and their cytoplasm is
                      filled with large refractile granules that stain red in blood smear
                      a- neutrophils
                      b- Eosinophilis
                      c- Basophiles
                      d- Lymphocytes
                      70- The cell which is responsible for antibody production is:
                      a- moncytes
                      b- T-lymphocytes
                      c- B-lymphocytes
                      d- Neutrophils

                      71- ……is a curved cell with sharp ends seen in haemoglobinopathies (HBS)
                      a- sickle cell
                      b- Spherocyte
                      c- Ovalocyte
                      d- Stomatocyte
                      72- All of the following is correct about sickle cell anemia except:
                      a- leg ulcers
                      b- Gall stones
                      c- Enlargement of spleen
                      d- Attacks of pain
                      73- ……….. is a single, large, rounded , dark , purple remnant of nucleus
                      a- Heinz body
                      b- Howeel-Jolly body
                      c- Pappenheimer body
                      d- Cabot ring
                      74- Agranulocyte:
                      a- neutrophil
                      b- Lymphocyte
                      c- Basophile
                      d- Eosinophil
                      75- Pica ( craving to eat unusual substance such as clay or ice) is one of the
                      symptoms of:
                      a- G6PD deficiency
                      b- Thalassemia
                      c- Megaloblastic anemia
                      d- Iron deficiency anemia
                      76- In…………….. there’s a decreased or absent hemosiderin in bone
                      marrow
                      a- sideroblastic anemia
                      b- Iron deficiency anemia
                      c- Megaloblastic anemia
                      d- Hemolytic anemia

                      77- Chloramphenicol may cause …………. anemia in long term therapy
                      a- iron deficiency
                      b- Vit B12 deficiency
                      c- folic acid deficiency
                      d- Aplastic anemia
                      78- ………. is the fluid (with anticoagulant) component of blood , it contains
                      salt & organic compounds:
                      a- plasma
                      b- Serum
                      c- Hemoglobin
                      d- Billirubin
                      79- Poikilocytosis is:
                      a- variation in red cell size
                      b- Variation in red cell color
                      c- Variation in red cell shape
                      d- non of the above
                      80- Red cell fragments:
                      a- echinocyte
                      b- Elliptocyte
                      c- Schistocyte
                      d- Stomatocyte
                      81- It is a defect of red cell member
                      a- Thalassemia
                      b- Sickle cell anemia
                      c- Hereditary spherocytosis
                      d- Megaloblastic anemia
                      82- All of the following is correct regarding spherocytosis except:
                      a- normocytic normochromic anemia
                      b- Decreased reticulocyte count
                      c- Raised plasma bilirubin
                      d- Increased osmotic fragility

                      83- Heinz bodies are seen in cases of
                      a- hereditary spherocytosis
                      b- Hereditary elliplocytosis
                      c- G6PD deficiency
                      d- sickle cell anemia
                      84- ………………is caused by substitution of amino acid (valine) instead of
                      glutamic acid at position No.#6 in the beta chain of hemoglobin
                      a- Hb-A
                      b- Hb-A2
                      c- Hb –F
                      d- Hb –S
                      85- Atrophy of the spleen is seen in cases of:
                      a- Thalassemia
                      b- Sickle cell anemia
                      c- G6PD deficiency
                      d- Hereditary elliplocytosis
                      86- iron deficiency lead to :
                      a- normocytic normochromic anemia
                      b- microcytic hypochromic anemia
                      c- macrocytic anemia
                      d- hemolytic anemia
                      87- Neurological symptoms are seen in cases of:
                      a- iron deficiency anemia
                      b- folic acid deficiency
                      c- Vit B12 deficiency
                      d- all of the above
                      88-Megaloplastic hematopoiesis is seen in cases of:
                      a- iron deficiency anemia
                      b- folic acid deficiency
                      c- Vit B12 deficiency
                      d- Vit C deficiency

                      89- Fanconi’s anemia is a type of :
                      a - vit B12 deficiency
                      b- aplastic anemia
                      c- Thalassemia
                      d- folic acid deficiency anemia
                      90- the most abundant leukocyte in a normal blood smear of adult is :
                      a- lymphocyte
                      b-Monocyte
                      c- Eosinophil
                      d- Neutrophil
                      91- The first line of defense against parasites:
                      a- neutrophils
                      b- Basophile
                      c- Eosinophil
                      d- Lymphocyte
                      92- …………….. play a role in immediate and delayed hypersensitivity:
                      a- monocyte
                      b- Lymphocyte
                      c- Eosinophil
                      d- Basophile
                      93- The largest leukocyte is :
                      a- neutrophils
                      b- Lymphocyte
                      c-Monocyte
                      d- Basophile
                      94- Cell which participate in cell mediated immunity:
                      a- monocyte
                      b- B- lymphocyte
                      c- T- lymphocyte
                      d- neutrophils

                      95- ………promotes blood clotting and help to prevent blood loss from
                      damaged blood vessels:
                      a- platelets
                      b-WBCs
                      c- RBCs
                      d- all of the above
                      96-antibody induced hemolytic disease in new born that is caused by blood
                      group incompatibility between mother and fetus:
                      a- hemolytic uremic syndrome
                      b- Erythroblastosis fetalis
                      c- Hereditary spherocytosis
                      d- Thromboloc thrombocytopenic purpurea
                      97- Young red blood cell with cytoplasmic RNA:
                      a- spherocyte
                      b- Reticulocyte
                      c- Stomatocyte
                      d- elliptocyte
                      98- Normal adult hemoglobin tetramer is:
                      a- 2 alpha : 2 gama
                      b- 2 alpha : 2 beta
                      c- 2 alpha : 2 delta
                      d- 2 beta : 2 gama
                      99- ……………represent 2-4 %of total leukocyte:
                      a- neutrophils
                      b- Basophile
                      c- Eosinophil
                      d-Monocyte
                      100- …………are small cytoplasmic fragment derived from megakaryocytic:
                      a- RBCs
                      b-WBCs
                      c- Platelet
                      d- non of the above

                      101 -………….. is the reduction in the amount of circulating hemoglobin ,
                      red blood cells or both:
                      a- polycythemia
                      b- Anemia
                      c- Hemophilia
                      d- Leucopenia
                      102- Thalassemia is :
                      a- microcytic anemia
                      b- Macrocytic anemia
                      c- Normocytic anemia
                      d- non of the above
                      103- Vit B12 deficiency lead to :
                      a- hemolytic anemia
                      b- Microcytic anemia
                      c- Normocytic anemia
                      d- Megaloblastic anemia
                      104- Lymphocyte represent ………….. of total leukocyte:
                      a- 20-40%
                      b- 50-70%
                      c- 2-8%
                      d- 1-5 %
                      105- Plumer- Vinson syndrome may be seen in cases of:
                      a- iron deficiency anemia
                      b- Vit B12 deficiency anemia
                      c- Aplastic anemia
                      d- folic acid deficiency anemia
                      106- peripheral, pale inclusions that push out the cell membrane and
                      composed of hemoglobin:
                      a- cabot ring
                      b- Pappenheimer body
                      c- Howell-Jolly body
                      d- Heinz body

                      107- it’s an acute hemolytic anemia occurring after the ingestion of broad
                      bean in individual with deficiency of G6PD :
                      a-thalassemia
                      b- Favism
                      c- Fanconi’s anemia
                      d- Cooley’s anemia
                      108-………………. is a multiple small , peripheral grape like purple clusters
                      of iron:
                      a- cabot ring
                      b- Heinz body
                      c- Howell-Jolly body
                      d- Pappenheimer body
                      109- the blood smear gives the physician information concerning:
                      a- morphology of RBCs and platelet
                      b- Presence of abnormal inclusion
                      c- Presence of immature cells
                      d- all of the above
                      110- hypersplenism is one of the causes of :
                      a- iron deficiency anemia
                      b- Hemolytic anemia
                      c- Aplastic anemia
                      d- Megaloblastic anemia
                      111- Increased reticulocytes count is seen in cases of:
                      a- hereditary spherocytosis
                      b- G6PD deficiency
                      c- Sickle cell anemia
                      d- all of the above
                      112- The antibody which can pass the placenta:
                      a- IgM
                      b- Ig G
                      c- IgD
                      d- Ig E

                      113- ………is an autoimmune disease in which there is an immune
                      destruction of the acid and pepsin secreating cells of the stomach:
                      a- fanconi’s anemia
                      b- cooley’s anemia
                      c-pernicious anemia
                      d- non of the above
                      114- All of these are laboratory features of aplastic anemia except:
                      a- pancytopenia
                      b-markedly hypocellular marrow
                      c- Increased reticulocyte count
                      d-Markedly increase in serum erythropoietin
                      115- Secondary granules of neutrophils contain:
                      a- elastase
                      b- Myeloperoxidase
                      c- Lysozyme
                      d- Histamine
                      116- Monocytes represent ………………. of total leukocyte:
                      a- 0-1%
                      b- 2-4%
                      c- 2-8%
                      d- 20-4 %
                      117- the reagent used for leukocyte count is :
                      a- citric acid
                      b- Acetic acid
                      c- Hydrochloric acid
                      d- Sulphoric acid
                      118- All are Prokaryotic cells except:
                      a- Fungi
                      b- Bacteria
                      c - Chlamydia
                      d- Mycoplasma

                      119- Viruses:
                      a- Contain only DNA or RNA
                      b - They Contain ribosome
                      c- Did not affected by antibiotics
                      d- a+c
                      120- All of these are essential structure except:
                      a- Nuclear body
                      b- Spores
                      c- Cell wall
                      d- Plasma Membrane
                      121- ______________ is giving the shape to the bacteria
                      a.- Cytoplasmic Membrane
                      b- Capsule
                      c- CellWall
                      d- All of the above
                      122- One of its functions is selective permeability
                      a- Cell wall
                      b- Plasma membrane
                      c- Capsule
                      d- Spores
                      123- They are responsible for Haemagglutination Phenomenon
                      a- Flagella
                      b- Fimbria
                      c- Capsule
                      d- Cell wall
                      124- Clostridium Tetani is:
                      a- Atrichous bacteria
                      b-Mono-trichous bacteria
                      c- Amphi-trichous bacteria
                      d- Peri-trichous bacteria

                      125- Short curved or straight rods, motile by single polar flagellum
                      a- spirochaeta
                      b- Vibrio
                      c- Escherichia
                      d- Lactobacillus
                      126- Small gram negative cocci, occur in pairs
                      a- staphylococcus
                      b- streptococcus
                      c- neisseria
                      d- non of the above
                      127- Transfer of genetic information from one bacterium to another by
                      bacteriophages is:
                      a. Transformation
                      b. Tansduction
                      c. Conjugation
                      d. Mutation
                      128- Salmonella are:
                      a- Obligatory Aerobic bacteria
                      b.- Obligatory Anaerobic bacteria
                      c- Facultative Anaerobic bacteria
                      d- Micro-aerophilic bacteria
                      129- According to pH, vibrio cholera is
                      a- Osmophilic bacteria
                      b- Basophilic bacteria
                      c- Acidophilic bacteria
                      d- Neutrophilic bacteria
                      130- Staphylococci are:
                      a- Atrichous bacteria
                      b- Mono-trichous bacteria
                      c- Amphi-trichous bacteria
                      d- Peri-trichous bacteria

                      131- During replication of DNA, copying errors may occur and this is called
                      a- Conjugation
                      b- Transduction
                      c- Transformation
                      d- Mutation
                      132- Obligatory Anaerobic bacteria
                      a- grow only in presence of oxygen
                      b- grow only in absence of oxygen
                      c- grow either in presence or absence of oxygen
                      d- grow in presence of oxygen traces and 5 – 10% CO2
                      133- Neutrophilic bacteria grow well at
                      a- pH 8.5 – 9.0
                      b- pH 7.2 – 7.4
                      c- pH 5.0 – 5.5
                      d- pH 2.5 – 3.0
                      134- The rate of cell death increase and bacterial growth stopped, this is
                      a- Adaptation phase
                      b- Exponential phase .
                      c- Stationary phase.
                      d- Decline Phase
                      135- Beta-hemolytic
                      a- Cause complete hemolysis of RBC’s
                      b- Cause chemical change of Hemoglobin in RBC’s
                      c- Do not cause hemolysis
                      d- None of them
                      146- Mesophilic bacteria grow at:
                      a- 37°C
                      b- 14°C
                      c- 60°C
                      d- 120°C

                      137- Circulation of Bacteria and its toxins in blood
                      a- Pyaemia
                      b- Toxemia
                      c.- Bacteremia
                      d- Septicemia
                      138- Disinfections that applied on living or injured tissues:
                      a- Sterilization
                      b- Antiseptic
                      c- Sanitation
                      d- Decontamination
                      139- Hot air oven is used for sterilization of
                      a- Glass
                      b- Rubber Gloves
                      c.- Plastic Syringes
                      d- Catheters
                      140- Disinfectant for superficial fungal infection
                      a- Phenol
                      b- Potassium permanganate
                      c- Chlorine
                      d- Hypochlorite compounds
                      141- Rifampin
                      a- inhibit cell wall synthesis
                      b- inhibit protein synthesis
                      c- inhibit folic acid pathway
                      d- inhibit mRNA synthesis
                      142- Transacetylase inactivate
                      a- aminoglycosides
                      b- chloramphenicol
                      c- penicillin
                      d- cephalosporins

                      143- The color of gram positive bacteria is
                      a- Yellow
                      b- Black.
                      c- Pink
                      d- Violet
                      144- Selective media for fungi
                      a- blood agar
                      b- Mac Conkey agar
                      c- Nutrient agar
                      d- Sabourand’s dextrose agar
                      145- Histoplasma is a :
                      a- Systemic mycosis
                      b- Sub – Cutaneous mycosis
                      c- Cutaneous mycosis
                      d- Superficial mycosis
                      146- All are asexual spores produced by mould except
                      a- Conidio – spores
                      b- Sporangio – spores
                      c- Endospores
                      d- Arthro – spores
                      147- They reproduce only by Asexual reproduction
                      a- Blastomycosis
                      b- Deutromycosis
                      c- Ascomycetes
                      d- Zygomycetes
                      148- The functions of cell wall is all of the following except:
                      a- Giving the shape to the bacteria
                      b- Carrying somatic antigen
                      c- Selective permeability and transport of solutes
                      d- Protect the bacteria from plasmolysis

                      149- Atrichous Bacteria are:
                      a- Bacteria contain one flagellum
                      b- Bacteria contain 2 flagella
                      c- Bacteria without flagella
                      d- Bacteria with a tuft of flagella
                      150- ________________ are essential for host cell attachment:
                      a- Flagella
                      b- Fimbria
                      c- Spores
                      d- Capsules
                      151- Irregular clusters of spherical cells:
                      a- Streptococcus
                      b- Staphylococcus
                      c- Lactobacillus
                      d- Escherichia
                      152- Clostridium Botulinum is:
                      a- Obligatory Aerobic Bacteria
                      b- Facultative Anaerobic Bacteria
                      c- Obligatory Anaerobic Bacteria
                      d- Micro- aerophilic Bacteria
                      153- Basophilic Bacteria grow well at:
                      a- pH 8.5 – 9.0
                      b- pH 7.2 – 7.4
                      c- pH 5.0 – 5.5
                      d- None of the above
                      154- Cells are divided at high & constant rate:
                      a- Decline Phase
                      b- Stationary Phase
                      c- Log Exponential Phase
                      d- Adaptation Phase
                      155- The dominant bacterial species in dental plaque are:
                      a- Coagulase Negative Staphylococci
                      b- Lactobacillus
                      c- Bacteroides
                      d- Streptococcus Sanguis

                      156- The spread of Pyogenic Bacteria in blood stream to different organs &
                      produce multiple abscess is:
                      a- Septicemia
                      b- Bacteremia
                      c-Toxemia
                      d- Pyaemia
                      157- Inhibit the growth of micro organisms
                      a- Bacteriostatic
                      b- Bactericidal
                      c- Fungicidal
                      d- Germicidal
                      158- To sterilize fluid damaged by heat:
                      a- Gaseous Sterilization
                      b- Heat Sterilization
                      c- Filtration
                      d- Ionizing Radiation
                      159- For water disinfection we use:
                      a- Hydrogen peroxide
                      b- Formaldehyde
                      c- Chlorine
                      d- Hypochlorite compounds
                      160- Mechanism of action of penicillin:
                      a- Block peptidoglycan synthesis
                      b- Inhibit peptidyglycan cross – linking
                      c- inhibit folic acid pathway
                      d- inhibit protein synthesis
                      161- Sulfonamides:
                      a- inhibit cell wall synthesis
                      b- inhibit protein synthesis
                      c- inhibit DNA synthesis
                      d- inhibit folic acid pathway

                      162- Acetylase inactivates:
                      a- B – Lactam antibiotics
                      b- Aminoglycosides
                      c- Cloramphenicol
                      d- All of the above
                      163- Ringworm disease is caused by
                      a- Zygomycetes
                      b- Ascomycetes
                      c- Blastomycosis
                      d- None of the above
                      164- For wet – mount technique we add:
                      a- NaoH
                      b- K oH
                      c- H2 O2
                      d- All of the above
                      165- They are transmitted by arthropods
                      a- Chlamydia
                      b- Spirochetes
                      c- Mycoplasma
                      d- All of the above
                      166- In the past they were listed as large viruses
                      a- Richettsia
                      b- Mycoplasma
                      c- Chlamydia
                      d- None of the above
                      167- Bacteria multiply by:
                      a- Replication cycle
                      b- Simple binary fission
                      c- Sexual reproduction
                      d- All of the above
                      168- It protects bacteria from antibiotics
                      a- Capsule
                      b- Cell membrane
                      c- Flagella
                      d- Fimbria

                      169- Vibro cholera is:
                      a-Mono –trichous bacteria
                      b- Atrichous bacteria
                      c- Lopho-trichous bacteria
                      d- Peri-trichous bacteria
                      170- Short rods, motile by peritrichous flagella
                      a- Spirochaeta
                      b- Lactobacillus
                      c- Escherichia coli
                      d- Vibrio
                      171- To take up soluble DNA fragments derived from other, closely related
                      species is:
                      a. Mutation
                      b. Transformation
                      c. Transduction
                      d. Conjugation
                      172- Tuberculosis are
                      a- micro-airophilic
                      b- Facultative anaerobic
                      c- Obligatory anaerobic
                      d- Obligatory aerobic
                      173- According to pH, Lactobacillus is
                      a- Neutrophlic bacteria
                      b- Acidophilic bacteria
                      c- Basophilic bacteria
                      d- None of the above
                      174- Bacteria without cellWall
                      a- Chlamydia
                      b- Rickettsia
                      c- Mycoplasma
                      d.- Spirochetes

                      175- Brucella Melitensis is
                      a- Obligatory aerobic bacteria
                      b- Obligatory anaerobic bacteria
                      c- Facultative anaerobic bacteria
                      d- Micro-aerophilic bacteria
                      176- Pseudomonas aeroginosa is
                      a- Peri-trichous bacteria
                      b- Lopho-trichous bacteria
                      c- Amphi-trichous bacteria
                      d- Monotrichous bacteria
                      177- Genetic information of bacteria is carried on
                      a- Messenger RNA
                      b- Transfer RN|A
                      c- Transcript RNA
                      d- Double – Stranded DNA
                      178- Thermophilic bacteria grow at
                      a- 60 – 80 °C
                      b- 0 - 20°C
                      c-. 37°C
                      d- 100 - 120°C
                      179- Acidophilic bacteria grow at
                      a- pH 7.2 – 7.4
                      b- pH 5.0 – 5.5
                      c- pH 8.5 – 9.0
                      d- None of the above
                      180- Mycoplasma is
                      a- Neutrophilic bacteria
                      b- Acidophilic bacteria
                      c- Basophilic bacteria
                      d- All of the above

                      181- It is the adaptation of bacteria to the fresh medium
                      a- Lag phase
                      b- Decline phase
                      c- Logarithmic Phase
                      d- Stationary phase
                      182- Bacteria which do not cause hemolysis is
                      a- Beta-Hemolytic
                      b- Alpha-Hemolytic
                      c- Gama Hemolytic
                      d- None of the above
                      183- Normal flora of Lower intestine are all of the following except:
                      a- Staphylococci
                      b- Diphtheroids
                      c- Shigella
                      d- Lactobacillus
                      184- Opportunistic pathogens are all of the following except:
                      a- Cause a disease when the host defense are suppressed.
                      b- Are normal flora of healthy body
                      c- Are greatly harmful
                      d- Do not invade the body or tissue.
                      185- For disinfection of mattresses :
                      a- Hot air oven
                      b- Autoclave
                      c- Ethylene Oxide
                      d- Hydrogen Peroxide
                      186- Pyschrophilic bacteria grow at:
                      a- 10٠°C
                      b- 6٠°C
                      c- 14°C
                      d- 37°C

                      187- Bacteria which contain chlorophyll
                      a- Heterotrophic bacteria
                      b- Autotrophic bacteria
                      c- Photosynthetic bacteria
                      d- All of the above
                      188- Tricophyton is one of
                      a- Yeast
                      b- Moulds
                      c- Dermatophyte
                      d- Dimorphic Fungi
                      189- Plastomyces is one of
                      a- Dermatophytes
                      b- Dimorphic Fungi
                      c- Yeast
                      d- Moulds
                      190- Color of gram negative bacteria is
                      a- Violet
                      b- Green
                      c- Red
                      d- Black
                      191- Acid Fast Bacteria
                      a- Salmonella
                      b- Shigella
                      c- M. Tuberculosis
                      d- E – Coli
                      192- Spherical or avoid cells occurring in chains
                      a- Staphylococci
                      b- Streptococci
                      c- Lactobacillus
                      d- Spiro chaeta

                      193- ………………carries the genetic information
                      a- the envelope
                      b- the capsid
                      c- the nucleic acid
                      d- the prion
                      194- ……………………may be seen under light microscope
                      a- rota virus
                      b- influenza virus
                      c- herps virus
                      d- pox virus
                      195- viruses may be:
                      a- monomorphic
                      b- pleomorphic
                      c- dimorphic
                      d- all of the above
                      196- viral capside is formed of:
                      a- protein
                      b- glycogen
                      c- lipoprotein bilayer
                      d- glycoprotein
                      197- class III in Baltimor classification is:
                      a- double stranded DNA viruses
                      b- single stranded DNA viruses
                      c- double stranded RNA viruses
                      d- single stranded RNA viruses
                      198- Hierarchial virus classification system use the following characters
                      except:
                      a- nature of nucleic acid
                      b- capside symmetry
                      c- diameter of viron & capside
                      d- virus molecular weight

                      199- in viral replication which is true:
                      a- penetration is the 1st step
                      b- assembly is the last step
                      c- relaease is the last step
                      d- all of the above
                      200- viron:
                      a- may be extracellular phase of virus
                      b- may be intracellular phase of virus
                      c- can grow and replicate
                      d- means “ virus – like “
                      201- pleomorphic viruses means :
                      a- virus which have constant shape
                      b- virus that may appear in 2 forms
                      c- virus that have not a constant morphology
                      d- virus that have spherical shape
                      202- vapor of gold is used in :
                      a- shadow casting technique
                      b- negative staining technique
                      c- positive staining technique
                      d- non of the above
                      203- direct diagnosis of virus :
                      a- ELISA
                      b- CFT
                      c- IFT
                      d- PCR
                      204- all of the following are required in cell culture except:
                      a- neutral PH
                      b- presence of buffer salts
                      c- presence of antibiotics
                      d- incubation at 20 C

                      205- all of the following are diagnostic molecular biological technique
                      except:
                      a- PCR
                      b- ELISA
                      c- nucleic acid hyperdization
                      d- DNA finger printing
                      206- PCR require all of the following except:
                      a- extracted DNA template
                      b- 2 specific primers
                      c- reation buffer
                      d- RNA polymerase
                      207- bacteriophage is :
                      a- virus that can be killed by antibiotic
                      b- virus that act like a bacteria
                      c- bacteria that act like a virus
                      d- virus that infect bacteria
                      208- all of the following viruses are transmitted by blood except:
                      a- HIV
                      b- HBV
                      c- HCV
                      d- herps virus
                      209-all of the following are RNA viruses except:
                      a- corona viridase
                      b- reoviridase
                      c- picorona viridase
                      d- pox viridase
                      210- penetration of naked virus is by :
                      a- fusion
                      b- endocytosis
                      c- translocation
                      d- all of the above

                      211- amniotic cavity inoculation is one type of virus culture in:
                      a- tissue wall
                      b- lab animals
                      c- embryonated egg
                      d- non of the above
                      212- …………….. is an invitro method amplification of a short sequence of
                      target DNA
                      a- PCR
                      b- hyberdization
                      c- finger printing
                      d- all of the above
                      213- nucleic acid hyberdization means:
                      a- probe anneling or binding with it’s complementary
                      segment of NA
                      b- fragmentation of nucleic acid
                      c- amplification of nucleic acid
                      d- non of the above
                      214- DNA hyberdization is performed by:
                      a- primers
                      b- DNA labeled probe
                      c- restriction endonuclease
                      d- non of the above
                      215-PCR starts with :
                      a- annealing
                      b- denaturation of DNA
                      c-extension of primers
                      d- non of the above
                      216- ……………….. is a piece of DNA fragment of a particular gene that can
                      bind specially with it’s complementary piece of DNA:
                      a- codon
                      b- probe
                      c- LCR
                      d- code

                      217- how many primers are used in PCR :
                      a- non
                      b- one
                      c- two
                      d- three
                      218- secreted from virus infected cell:
                      a- C-reactive protein
                      b- complement
                      c- interferons
                      d- lysozyme
                      219- it’s a process of coating the foreign cell by antibodies :
                      a- chemotaxis
                      b- adherence
                      c- opsonization
                      d- endocytosis
                      220- the only Ig that can cross the placenta:
                      a- IgE
                      b- IgD
                      c- IgM
                      d- IgG
                      221- the predominant Ig in primary immune response:
                      a- IgM
                      b- Ig A
                      c- Ig G
                      d- Ig D
                      222- the predominant Ig in body secretion:
                      a- IgM
                      b- Ig A
                      c- Ig G
                      d- Ig D

                      223- it’s a cytophilic Ab
                      a- IgM
                      b- Ig E
                      c- Ig G
                      d- Ig D
                      224- it’s the main host defense against helminthes infestations:
                      a- Ig E
                      b- Ig A
                      c- Ig G
                      d- Ig D
                      225- Antibodies are secreted from:
                      a- B-lymphocyte
                      b- T- lymphocyte
                      c- monocyte
                      d-Macrophage
                      226-immunity through placenta is:
                      a- natural active immunity
                      b- Artificial active immunity
                      c- Natural passive immunity
                      d- Artificial passive immunity
                      227- one of the primary lymphoid organs:
                      a - spleen
                      b- Tonsils
                      c- Thymus gland
                      d- Lymph nodes
                      228- Its a memory of T cell mediated immunity:
                      a-T – helper
                      b- T suppressor
                      c- T- cytotoxic
                      d- T effector

                      229- all are character of primary immune response except:
                      a- stimulate native B- cells
                      b- short duration
                      c- high level of Ab
                      d- low affinity of Abs to Ags
                      230- they are cytokines that regulate the interaction between lymphocyte and
                      other leukocyte:
                      a- interferon
                      b- Growth factor
                      c- Tumor necrosis factor
                      d- Interleukins
                      231- Malignant tumor in epithelial cells:
                      a- leukemia
                      b- Carcinoma
                      c- Sarcoma
                      d- non of the above
                      232- Kaposi’s sarcoma is caused by:
                      a- hepatitis B virus
                      b- Epstein Barr virus
                      c- Cytomegalo virus
                      d- Human papilloma virus
                      233- Reagenic Ab:
                      a- IgM
                      b- Ig A
                      c- Ig G
                      d- Ig E
                      234- Tuberculin test is :
                      a- type I hypersensitivity
                      b- type II hypersensitivity
                      c- type III hypersensitivity
                      d- type VI hypersensitivity

                      235- Formation of auto-Abs ( IgG) to acetylcholine receptors:
                      a- multiple sclerosis
                      b- Graves’s disease
                      c-Myasthenia gravis
                      d- Rheumatoid arthritis
                      236- Auto –Abs against Ags on the myelin sheath:
                      a- Hashimoto’s thyroditis
                      b- Graves’s disease
                      c- Myasthenia gravis
                      d- Multiple sclerosis
                      237- Anti nuclear Abs found in case of
                      a- SLE
                      b- IMN
                      c- CML
                      d- AML
                      238- If the rejection to kidney grafts is happened after 7 days this is:
                      A- Hyper acute rejection
                      b- Accelerated rejection
                      c- Acute rejection
                      d- Chronic rejection
                      239- The test which determine the presence of Ab to HIV Ags(p24,P31)is:
                      a- ELISA test
                      b- immunfluoresence test
                      c- western blot test
                      d- P24 Ag test
                      240- Vaccination is:
                      a- natural active immunity
                      b- Artificial active immunity
                      c- Natural passive immunity
                      d- Artificial passive immunity

                      241- Lysozyme found in high concentration:
                      a- urine
                      b- Sweat
                      c- Tears
                      d- CSF
                      242- ………….. inhibit virus replication:
                      a- acute phase protein
                      b- Lysozyme
                      c- Complement
                      d- Interferon
                      243- it’s a foreign substance of low molecular weight which can not induce
                      specific immune response unless it’s conjugated with a carrier:
                      a- antigen
                      b- Immunogene
                      c- Hatpin
                      d- Adjuvant
                      244-the main Ig in secondary immune response:
                      a- Ig A
                      b- Ig G
                      c- IgD
                      d- IgM
                      245- Wassermantest is done for the diagnosis of :
                      a- T.B
                      b- Syphilis
                      c- Diphtheria
                      d- all of the above
                      246- Elek test is done for the diagnosis of:
                      a- T.B
                      b- Syphilis
                      c- Diphtheria
                      d- Gonorrhea

                      247- thyrotoxin is secreted from:
                      a- pituitary gland
                      b- Thyroid gland
                      c- Suprarenal gland
                      d- non of the above
                      248-they are material that enhance the normal immune response non
                      specifically:
                      a- antigen
                      b- Hapten
                      c- Adjuvant
                      d- Mitogen
                      249- they increase the mitotic activity of lymphocyte:
                      a- immunogens
                      b- Adjuvant
                      c- Hapten
                      d- Mutagen
                      250- it’s the most efficient Ig in complement fixation:
                      a- IgM
                      b- Ig A
                      c- Ig G
                      d- Ig E
                      251- complement concentration increased in:
                      a- nephritis
                      b- Serum sickness
                      c- Carcinoma
                      d-Malaria
                      252- the acceptable minimum concentration of heamoglobin for the donor is:
                      a- 12-5 g/dl
                      b- 10-5 g/dl
                      c- 9.5 g/dl
                      d- 15.5 g/dl

                      253- all of these are adverse donor reaction except :
                      a- syncope
                      b- convulsions
                      c- hypertension
                      d- nausea and vomiting
                      254- the bleeding room should be equipped with the following injections
                      ……. except:
                      a- adrenaline
                      b- atropine
                      c- dexamethasone
                      d- calcium gluconate
                      255- the ****f life of deglycerolized blood is:
                      a- 24 hours
                      b- 48 hours
                      c- 72 hours
                      d- 12 hours
                      256- the platelet are stored at :
                      a- 2 : 6 c ْ
                      b- 20 : 26 c ْ
                      c- 20 : 50 c ْ
                      d- 120 : 180 c ْ
                      257- major cross match:
                      a- donor’s cells + patient serum
                      b- donor’s serum + patient’s cell
                      c- donor’s cells + donor’s serum
                      d- patient’s cells + patient’s serum
                      258- cold agglutination syndrome is due to :
                      a- IgG
                      b- IgM
                      c- IgA
                      d- Ig D

                      259- the antibody usually responsible for PCH is:
                      a- anti A
                      b- anti B
                      c- anti Po
                      d- anti D
                      260- the worm autoimmune hemolytic anemia is due to :
                      a- IgM
                      b- IgA
                      c- Ig G
                      d- Ig D
                      261- HLA antigen is controlled by a region on chromosome No.:
                      a- 9
                      b- 6
                      c- 12
                      d- 3
                      262- the blood group individuals which are found to be resistance to a west
                      African malaria parasite:
                      a- kell
                      b- kidol
                      c- duffy
                      d- Lutheran
                      263- the color of anti-sera A:
                      a- yellow
                      b- colorless
                      c- blue
                      d- green
                      264- the color of anti-sera B is:
                      a- red
                      b- blue
                      c- yellow
                      d- colorless

                      265- detecting ABO grouping by using the cells of the patient is:
                      a- reverse grouping
                      b- forward grouping
                      c- backward grouping
                      d- non of the above
                      266- detecting ABO grouping using the serum of the patient is
                      a- reverse grouping
                      b- forward grouping
                      c- backward grouping
                      d- non of the above
                      267- patient with anti A antibodies in his serum , his blood group is :
                      a- group A
                      b- group B
                      c- group AB
                      d- groupO
                      268-the red cells of a patient with neither A nor B antigen on the surface , his
                      blood group is :
                      a- group A
                      b- group B
                      c- group AB
                      d- groupO
                      269- cryoprecipitated is used for the treatment of :
                      a- hemophilia A
                      b- anemia
                      c- leukemia
                      d- thrombocytopenia
                      270- universal donor is :
                      a- group A
                      b- group B
                      c- group AB
                      d- groupO

                      271-universal recipient is :
                      a- group A
                      b- group B
                      c- group AB
                      d- groupO
                      272- fresh frozen plasma is used for the treatment of the following…….
                      except:
                      a- DIC
                      b- vit K deficiency
                      c- clotting factor deficiency
                      d- protein replacement
                      273- the indication for exchange transfusion :
                      a- disseminated intravascular coagulation
                      b- immune thrombocytopenic purpura
                      c- hemolytic disease of newborn
                      d- vit K deficiency
                      274- a case of anemia with coronary disease must be treated with:
                      a- whole blood
                      b- packed red cells
                      c- fresh frozen plasma
                      d- cryoprecipitate
                      275- the storage time of acid citrate dextrose (ACD) is:
                      a- 42 days
                      b- 21 days
                      c- 35 days
                      d- 15 days
                      276- the storage time of saline adenine glucose – mannitol (SAGM) is:
                      a- 42 days
                      b- 21 days
                      c- 35 days
                      d- 15 days

                      277- to collecte blood from the donor , you have to apply the blood pressure
                      cuff on his arm and raise the pressure to:
                      a- 150 – 180 mmHg
                      b- 15 -16 mmH
                      c- 50 -60 mmHg
                      d- non of the above
                      278- all of these are transfusion transmitted disease except:
                      a- hepatitis B
                      b- hepatitis C
                      c- hepatitis A
                      d- AIDS
                      279- the donor is permanently deferred if gives a positive history of :
                      a- German measles
                      b- antidiphtheria vaccine
                      c- epilepsy
                      d- tattooing
                      280- the blood donor must not be less than:
                      a- 30 years
                      b- 18 years
                      c- 40 years
                      d- 25 years
                      281- the clinical significance of Rh antigen :
                      a- blood transfusion
                      b- hemolytic disease of newborn
                      c- medico legal
                      d- all of the above
                      282- in cross matching we use:
                      a- albumin
                      b- antihuman globulin
                      c- papain enzyme
                      d- all of the above

                      283- the temperature of the blood refrigerator is:
                      a- 6 – 9C ْ
                      b- 2 – 6 C ْ
                      c- 9 – 15 C ْ
                      d- 20 – 25 C ْ
                      284- biochemical changes in the stored blood RBCs:
                      a- RBCs change it’s shape
                      b- osmotic fragility is increased
                      c- loss of red cell membrane lipids
                      d- all of the above
                      285- the red cells in frozen state can be stored for years by addition of:
                      a- oxygen
                      b- hydrogen
                      c- nitrogen
                      d- glycerol
                      286- dark red card on blood bag means:
                      a- group O +ve
                      b- group A +ve
                      c- group B +ve
                      d- group AB +ve
                      287- panel test is done to know:
                      a- viruses on the serum
                      b- antigen on the serum
                      c- antibodies on the serum
                      d- all of the above
                      288- incomplete antibody :
                      a- IgM
                      b- IgA
                      c- Ig G
                      d- Ig D

                      289- the anticoagulant used for blood coagulation and platelet function
                      studies:
                      a- EDTA
                      b- heparin
                      c- trisodium citrate
                      d-all of the above
                      290- the preferred anticoagulant for blood cell count :
                      a- EDTA
                      b- heparin
                      c- trisodium citrate
                      d-all of the above
                      291- haymen’s solution is used for:
                      a- RBCs counting
                      b-WBCs counting
                      c- platelet counting
                      d- Hb concentration
                      292-the most commonly used diluting fluid for RBCs count:
                      a- Drakin’s solution
                      b- Turk’s solution
                      c- Normal saline
                      d- glacial acetic acid
                      293- Drabkin’s solution is used for:
                      a- RBCs counting
                      b-WBCs counting
                      c- platelet counting
                      d- Hb concentration
                      294-in hemoglobin determination by using cyanmethemoglobin method . we
                      measure the absorbency in spectrophotometer at a wave length of:
                      a- 504 nm
                      b- 405 nm
                      c- 540 nm
                      d- 450 nm

                      295- the ratio of the volume of cellular elements to that of whole blood is:
                      a- MCV
                      b-MHC
                      c- MCHC
                      d- HCT
                      296- the normal value of mean corpuscular volume is:
                      a- 27 – 32 pg
                      b- 80 – 90 fl
                      c- 33 – 38%
                      d- non of the above
                      297- (PCV/ RBCs count ) x 10 =
                      a-MCH
                      b- MCHC
                      c-MCV
                      d- HCT
                      298- thoma pipette is used for :
                      a- RBCs counting
                      b-WBCs counting
                      c- platelet counting
                      d- Hb concentration
                      299- glacial acetic acid is used for :
                      a- RBCs counting
                      b-WBCs counting
                      c- platelet counting
                      d- reticulocyte counting
                      300- for counting platelet we use:
                      a- ammonium oxalate
                      b- mercury chloride
                      c- sodium carbonate
                      d- ammonium hydroxide

                      301- normal adult have a reticulocyte count of :
                      a- 2.5 % - 6.5 %
                      b- 0.5 % - 2%
                      c- 5 % - 10%
                      d- non of the above
                      302- brilliant cresyl blue is used for:
                      a- RBCs counting
                      b-WBCs counting
                      c- platelet counting
                      d- reticulocyte counting
                      303- Westergren’s pipette is used for :
                      a- HCT estimation
                      b- ESR estimation
                      c- Hb determination
                      d-WBCs counting
                      304-Lowenstein-Jensen media is used for the isolation for:
                      a- neisseria gonorrhea
                      b- mycobacterium tuberculosis
                      c- haemophilus influenza
                      d- staphylococcus aureus
                      305- Hekton-Enteric agar is the selective media for:
                      a- salmonella
                      b- streptococcus
                      c- staphylococcus
                      d- all of the above
                      306- the selective media for isolation of fungi is:
                      a- S-S agar
                      b- XLD agar
                      c- sabouraud glucose agar
                      d- Hekton-Enteric agar

                      307- gram positive rods with Chinese letter appearance:
                      a- mycobacterium tuberculosis
                      b- corynebacteria diphtheria
                      c- clostridium tetani
                      d- staphylococcus pneumonia
                      308- E lek test is done to diagnose :
                      a- streptococcus
                      b- staphylococcus aureus
                      c- clostridium tetani
                      d- corynebacteria diphtheria
                      309- all are lactose fermenter except:
                      a- E-COLI
                      b- proteus
                      c- klebsiella
                      d- enterobacter
                      310- produce pale colonies onMacConkey’s agar and have tendency to
                      swarm on blood agar:
                      a- salmonella
                      b- shigella
                      c- klebsiella
                      d- proteus
                      311- lactose frementer gram negative bacilli with mucoid growth:
                      a- salmonella
                      b- shigella
                      c- klebsiella
                      d- proteus
                      312-non lactose fermenter gram negative bacilli produce H2S :
                      a- salmonella
                      b- shigella
                      c- klebsiella
                      d- proteus

                      313- Widal test is done for diagnosis of :
                      a- salmonella
                      b- shigella
                      c- E-coli
                      d- klebsiella
                      314- the causative agent of enteric fever:
                      a- salmonella
                      b- shigella
                      c- klebsiella
                      d- proteus
                      315- the most common cause of urinary tract infection :
                      a- E-coli
                      b- salmonella
                      c- shigella
                      d- streptococcus
                      316- an important cause of diarrhea in infant:
                      a- staphylococcus
                      b- E-coli
                      c- salmonella
                      d- shigella
                      317- the most common causative agent for peptic ulcer :
                      a- campylobacter
                      b- H-pylori
                      c- V-cholera
                      d- all of the above
                      318- TCBS is the selective media for isolation of:
                      a- H-pylori
                      b- V-cholera
                      c- E-coli
                      d- H influenza

                      319- gram negative bacilli strict aerobes grows on simple media producing a
                      characteristic greenish pigment:
                      a- campylobacter
                      b- pseudomonas
                      c- pasterulla
                      d- bordetella
                      320- the bacteria which cause scarlet fever:
                      a- staphylococcus
                      b- streptococcus
                      c- salmonella
                      d- shigella
                      321- antistreptolysin O titer (ASO) is done for the diagnosis of:
                      a- group A streptococcus
                      b- group B streptococcus
                      c- staphylococcus aureus
                      d- staphylococcus albus
                      323- Loffler’s serum is used for isolation of:
                      a- anthrax
                      b- clostridium
                      c- diphtheria
                      d- T.B
                      324- gas gangrene is caused by:
                      a- clostridium tetani
                      b- clostridium botulinum
                      c- clostridium welchii
                      d- non of the above
                      325- Bacillary dysentery is caused by:
                      a- salmonella
                      b- shigella
                      c- cholera
                      d- all of the above

                      326- the cause of plague:
                      a- Y-enterocolitica
                      b- Y-pestis
                      c- Y pseudotuberculsois
                      d- non of the above
                      327- treponema palladium is the cause of :
                      a- T.B
                      b- gonorrhea
                      c- syphilis
                      d- AIDS
                      328- the venereal disease research laboratory test (VDRL|) is done for
                      diagnosis of:
                      a- T.B
                      b- gonorrhea
                      c- syphilis
                      d- AIDS
                      329- Trachoma is caused by:
                      a- mycoplasma
                      b- chlamydiae
                      c-richettsia
                      d- mycobacteria
                      330- ………….. are substance produced by specialized cells of the body and
                      carried by blood stream where it affect other specialized cells:
                      a- vitamins
                      b- enzymes
                      c- isoenzyme
                      d- hormones
                      331- ……………is a protein which catalyse one or more specific biochemical
                      reaction and not consumed during the reaction:
                      a- enzymes
                      b- hormones
                      c- vitamins
                      d- proteins

                      332- …………is required in the hepatic synthesis of prothrombin and the
                      blood clotting factors and it’s deficiency is observed in newborn infants:
                      a- vitamin E
                      b- vitamin A
                      c- vitamin K
                      d- vitamin D
                      333- there are enzymes that catalyze the same reaction but differ in there
                      physical properties:
                      a- vitamins
                      b- adjuvents
                      c- isoenzyme
                      d- hormones
                      334- it’s functionis to maintain adequate serum level of calcium:
                      a- vitamin E
                      b- vitamin A
                      c- vitamin K
                      d- vitamin D
                      335-the inhibitor and substrate bind at different sites on the enzyme this type
                      of inhibition is called:
                      a- competitive inhibition
                      b- non competitive inhibition
                      c- surface recognition
                      d- product concentration
                      336- ……….. found in cartilage consist of a core protein to which the linear
                      carbohydrates chain are covalently attached:
                      a- glycoprotein
                      b- proteoglycan
                      c- link protein
                      d- hyaluronic acid
                      337- ……………is synthesized only by micro-organism , it’s not present in
                      plants but present in liver , it’s deficiency leads to pernicious anemia:
                      a- vit C
                      b- vit B12
                      c-vit B1
                      d- vit B2

                      338- the brown color of the stool is due to the presence of:
                      a- urobilinogen
                      b- urobilin
                      c- porphyrin
                      d- bilirubin
                      339- the degradation of heme takes place in the…………. particularly in the
                      liver and spleen
                      a- reticulocytes
                      b- erythrocytes
                      c- reticuloendothelial cells
                      d- non of the above
                      340- increased Hb destruction , the liver is unable to cup the greater load of
                      pigment and bilirubin level well rises this is called :
                      a- hepatogenous jaundice
                      b- hemolytic jaundice
                      c- obstructive jaundice
                      d- non of the above
                      341- …………plays a role in visual cycle it’s deficiency leads to night
                      blindness, β-carotene is the major precursor of this vitamin in human:
                      a- vitamin E
                      b- vitamin A
                      c- vitamin K
                      d- vitamin D
                      342- a large percentage of the……….. requirement in humans is supplied by
                      intestinal bacteria:
                      a- biotin
                      b- niacin
                      c- folic acid
                      d- thiamin
                      343- ………….. are organic compounds required by the body in trace
                      amount , can’t be synthesized by humans , but supplied by diet:
                      a- enzymes
                      b- vitamins
                      c- hormones
                      d- proteins

                      344- it facilitate the absorption of iron by reducing it to ferrus state in the
                      stomach and it’s deficiency result in scurvy:
                      a- vit C
                      b- vit B
                      c- vit D
                      d- vit A
                      345- it’s function is to transport oxygen from the lung to the tissue:
                      a- haptoglobin
                      b- hemoglobin
                      c- bilirubin
                      d- myoglobin
                      346- ………….. act as an antioxidant and it’s deficiency cause liver
                      degeneration:
                      a- vit E
                      b- vit A
                      c- vit K
                      d- vit D
                      347- it’s caused by liver parenchyma damage , the excretion of bile greatly
                      decreased and the concentration of bilirubin in the blood rise :
                      a- hemolytic jaundice
                      b- hepatogenous jaundice
                      c- obstructive jaundice
                      d- non of the above
                      348- ………play an essential role in body metabolism , a deficiency or excess
                      may lead to serious dearrangement of body function:
                      a- enzymes
                      b- hormones
                      c- vitamins
                      d- isoenzymes
                      349- …………. will interfere with the chemical determination of bilirubin ,
                      giving high variable results:
                      a- hemolysis
                      b- hemoglobin
                      c- vitamins
                      d- hormones

                      350- in hemolytic jaundice there will be increase……………. in serum:
                      a- direct bilirubin
                      b- indirect bilirubin
                      c- total bilirubin
                      d- all of the above
                      351- the inhibitor binds reversibly to the same site on the enzyme that the
                      substrate normally occupy , this type of inhibition is called :
                      a- competitive inhibition
                      b- non competitive inhibition
                      c- surface recognition
                      d- product concentration
                      352- regulation of blood glucose level can be achieved by :
                      a- hormonal mechanism
                      b- hepatic and renal mechanism
                      c- (a) & (b)
                      d- non of the above
                      353- insulin is a hormone secretes by :
                      a- α cell of islet of langerhans in pancreas
                      b- β cell of islet of langerhans in pancreas
                      c- suprarenal cortex
                      d- non of the above
                      354- cholesterol is a component of all cell membrane and it’s the precursor
                      of :
                      a- bile acid
                      b- steroid hormones
                      c- vit D
                      d- all of the above
                      355- anti diuretic hormone ( ADH) secretion is controlled by:
                      a- rennin angiotensin
                      b- plasma osmlality
                      c- (a) & (b)
                      d- non of the above

                      356- the electrophoresis is based on differential migration of :
                      a- charged particles
                      b- uncharged particles
                      c- molecular weight
                      d- (a) & (b)
                      357- acid base balance is regulated by :
                      a- oxygen concentration
                      b- hydrogen ion concentration
                      c- nitrogen ion concentration
                      d- (a) & (b)
                      358- it’s an increase in hydrogen ion concentration of the blood:
                      a- acidosis
                      b- alkalosis
                      c- acid base balance
                      d- (a) & (b)
                      359- over production of acid associated with :
                      a- diabetes mellitus
                      b- lactic acidosis
                      c- methanol poisoning
                      d- all of the above
                      360- serum bicarbonate is decreased in:
                      a- respiratory acidosis
                      b- metabolic acidosis
                      c- renal tubular acidosis
                      d- all of the above
                      361- chronic deficiency in dietry calcium can lead to :
                      a- anemia
                      b- bronchial asthma
                      c- osteoporosis
                      d- non of the above

                      362- …………is due to decrease blood CO2:
                      a- metabolic acidosis
                      b- respiratory acidosis
                      c- respiratory alkalosis
                      d- metabolic acidosis
                      363- …………. is the most important factor affecting body sodium content:
                      a- aldosteron secretion
                      b- antidiuretic hormone
                      c- testosterone
                      d- all of the above
                      364- haemosiderosis is :
                      a- increase iron store
                      b- decrease iron store
                      c- increase hemoglobin
                      d- decrease hemoglobin
                      365- if there’s a mixture of protein ( colloids) and salt( crystalloid) they can
                      be separated by :
                      a- precipitation
                      b- dialysis
                      c- chromatography
                      d- electrophoresis
                      366- the predominant cation in intracellular fluid is :
                      a- sodium
                      b- potassium
                      c- calcium
                      d- phosphorus
                      367- metabolic acidosis is due to :
                      a- failure to secret acid
                      b- bronchial asthma
                      c- loss of bicarbonate
                      d- (a) &(c)

                      368- high level of plasma ferritin may occur due to :
                      a- inflammatory condition
                      b- malignant disease
                      c- liver disease
                      d- all of the above
                      369- gonadal hormones estimation is important in :
                      a- detection of ovulation
                      b- assessment of amenorrhea
                      c- evaluation of delayed puberty
                      d- all of the above
                      370- the secretion of gonadal hormone is controlled by :
                      a- LH
                      b- FSH
                      c- TSH
                      d- (a) & (b)
                      371- the intensity of the color is directly proportional to the………. of the
                      analyte in the solution:
                      a- dilution
                      b- contamination
                      c- concentration
                      d- observation
                      372- the…………contain information of any health or safety rich associated
                      with use or exposure to hazardous chemicals:
                      a- MSDS
                      b- NFPA
                      c- POLT
                      d- OSHA
                      373- instraument used to measure color changes in the labs:
                      a- microscope
                      b- centrifuge
                      c- photometer
                      d- all of the above

                      374- the color coded signs used to identify flammable chemicals:
                      a- blue
                      b- yellow
                      c- white
                      d- red
                      375- quality assurance includes :
                      a- personal orientation
                      b- laboratory documentation
                      c- knowledge of laboratory istraumentation
                      d- all of the above
                      376- the laboratory procedure manual include:
                      a- patient preparation
                      b- specimen collection & processing
                      c- specimen preservation , storage & transport
                      d- all of the above
                      377- the principal of reflectance photometer
                      a- measure the amount of light that pass through the solution
                      b- measure the amount of light that the solution absorbs
                      c- (a) & (b)
                      d- non of the above
                      378- the blood cell counter include :
                      a- aperture impedence cell counter
                      b- Geiger counter
                      c- microscopes
                      d- all of the above
                      379- it is mession is to save lives , prevent injuries , and protect health of all
                      workers in the lab. :
                      a- MSDS
                      b- NFPA
                      c- POLT
                      d- OSHA

                      380-……………. requires 3 hours at 140 Cْ or 1 hour at 160 C ْ for complete
                      sterilization
                      a- hot air oven
                      b- autoclave
                      c- filteration
                      d- all of the above
                      381- arterial blood samples are essential to do :
                      a- CBC
                      b- urea
                      c-blood glucose
                      d- blood gas analysis
                      382- vaccum tubes with green stopper contain:
                      a- EDTA
                      b- sodium citrate
                      c- heparin
                      c- no anticoagulant
                      383- serum separator tube is all of the following except:
                      a- contain gel that separate serum from cells during centrifugation
                      b- contain clot activator to speed clot formation
                      c- has red & black mottled top stopper
                      d- used for coagulation tests
                      384- for phlepotomy we use all of the following except:
                      a- the hypodermic needle & syringe
                      b- the vaccum tube system
                      c- the monolet lancets
                      d- the winged infusion set
                      385- lab. equipment should be cleaned and disinfected with :
                      a- hypochlorite
                      b- formaldehyde
                      c- glutaraldehyde
                      d- (b) & (c)

                      386- any blood split in the lab should be immediately swabbed with :
                      a- hypochlorite
                      b- alcohol
                      c- soap
                      d- water
                      387- CBC is performed using:
                      a- serum
                      b- well mixed EDTA whole blood
                      c- plasma
                      d- non of the above
                      388- which tube should be filled first in blood collection:
                      a- tubes with anticoagulant
                      b- tube without anticoagulant
                      c- tubes for blood culture
                      d- non of the above
                      389- the monojector is designed to be used with :
                      a- the monolet lancet
                      b- tenderlett
                      c- tenderfoot
                      d- non of the above
                      390- the site of choice for capillary puncture in newborns is :
                      a- the earlobe
                      b- middle finger
                      c- the big toe
                      d- the lateral medial planter heel surface
                      391- M-tuberculsis bacilli stain with :
                      a- gram stain
                      b- Zheil Nelson stain
                      c- Gimesa stain
                      d- all of the above

                      392- AIDS is transmitted through :
                      a- food
                      b- blood
                      c- semen
                      d- (b) & (c)
                      393- bacteria which cause syphilis:
                      a- Neisseria gonorrhea
                      b- Viencent angina
                      c- Treponema palladium
                      d- Yersinia pestis
                      394- to diagnose syphilis:
                      a- RPR
                      b- VDRL
                      c-Wasserman
                      d- all of the above
                      395- streptococci secret:
                      a- streptolysinO
                      b- streptolysin S
                      c- streptokinase
                      d- all of the above
                      396- disease caused by streptococci:
                      a- scarlet fever
                      b- purperal sepsis
                      c- rheumatic fever
                      d- all of the above
                      397- staphylococci secrets:
                      a- coagulase enzyme
                      b- fibrinolysin
                      c- hyaluronidase
                      d- all of the above

                      398- gram positive bacilli:
                      a- Klebsilla
                      b- Salmonella
                      c- Proteus
                      d- C-diphtheria
                      399- meningeococcal meningitis is transmitted by :
                      a- food
                      b- droplet
                      c- touch
                      d- all of the above
                      400- dark field microscopy is used to diagnose :
                      a- T.B.
                      b- syphilis
                      c- gonorrhea
                      d- AIDS
                      401- it cause food poisining with flacid paralysis:
                      a- clostridium tetani
                      b- clostridium welchii
                      c- clostridium botulinium
                      d- all of the above
                      402- the infective stage of plasmodium vivax :
                      a-merozoites
                      b- sporozoites
                      c- schizont
                      d- trophozoite
                      403- Pirenella conica snail is the intermediate host of:
                      a- schistosoma haematobium
                      b- fasciola hiptica
                      c- heterphyes heterophyes
                      d- diphyllobothrium latum

                      404- Bulinus truncates snail is the intermediate host of :
                      a- fasciola hepatica
                      b- fasciola gigantica
                      c- schistosoma haematobium
                      d- schistosoma mansonii
                      405- to isolate meningiococci we have to culture the sample on:
                      a- Bordet Gengou
                      b- modified Thayer martin media
                      c- Lowenstein Jensen media
                      d- all of the above
                      406- to isolate fungi :
                      a- Brain-Heart infusion media
                      b- tissue culture
                      c- Lowenstein –Jensen media
                      d- chocolate agar
                      407- to isolate H- influenza:
                      a- blood agar
                      b- chocolate agar
                      c- mac Conkey media
                      d- all of the above
                      408- the bacteria which cause pseudomembrainous conjunctivitis :
                      a- N.gonorrhea
                      b- C. diphtheria
                      c- staphylococcus
                      d- Chlamydia
                      409- the best sample to diagnose meningitis :
                      a- blood
                      b- sputum
                      c- CSF
                      d- urine

                      410- used to stain Chlamydia
                      a- gram stain
                      b- giemsa stain
                      c- wright stain
                      d- all of the above
                      411-……….. is used as transport medium for sample in which cholera is
                      suspected
                      a- Cary-Blair media
                      b- Stuart media
                      c- Alkaline peptone water
                      d- glycerol
                      412- the color of XLD medium:
                      a- green
                      b- red
                      c- yellow
                      d- blue
                      413- CIN medium is used to isolate:
                      a- E.coli
                      b- Vibrio cholera
                      c- yersinia
                      d- salmonella
                      414- to make wet mount preparation:
                      a- 10 % KOH
                      b- 10 % Na OH
                      c- 10 % Na CO3
                      d- 10% Na Cl
                      415- we do wet mount preparation for vaginal smears to diagnose:
                      a- T.vaginalis
                      b- N.gonorrhea
                      c- streptococci
                      d- staphylococci

                      416- to isolate viruses:
                      a- Loeffler media
                      b- tissue culture
                      c- Bordet –Gengou media
                      d- Brain- Heart infusion
                      417- to diagnose whooping cough :
                      a- Bordet –Gengou media
                      b- Lowenstein –Jensen media
                      c- modified Thayer martin media
                      d- New York city agar
                      418- to diagnose systemic infection we do :
                      a- urine culture
                      b- CSF culture
                      c- blood culture
                      d- sputum culture
                      419- we give no growth for blood culture after:
                      a- 1 week
                      b- 8 weeks
                      c- 6 weeks
                      d- 3 weeks
                      420- to dissolve mucous in sputum sample :
                      a- 10% NaOH
                      b- 30 % NaOH
                      c- 10 % KOH
                      d- 10% NaCl
                      421- mutualism means:
                      a- one partener benefits , other unaffected
                      b- both partner benefit
                      c- one partner benefit , other damaged
                      d- living together

                      422- Commensalisms means:
                      a- living together
                      b- one partner benefit , other damaged
                      c- both partner benefit
                      d- one partner benefits , other unaffected
                      423- Balantidium coli moves by:
                      a- flagella
                      b- cilia
                      d- pseudopod
                      d- all of the above
                      424- Mouth inhabitant:
                      a- Trichomonas hominis
                      b- Trichomonas tenax
                      c- Trichomonas vaginalis
                      d- giardia lamblia
                      425- Transmitted by sexual intercourse:
                      a- toxoplasma
                      b- giardia lamblia
                      c- Trichomonas vaginalis
                      d- all of the above
                      426- Intermediate host of Trypansom:
                      a- triatoma megista
                      b- sand fly
                      c- tse tse fly
                      d- anopheles
                      427- The cause of chaga's disease:
                      a- trypanosoma gambiense
                      b- trypansoma rhodesiense
                      c- trypansoma cruzi
                      d- leishmania braziliense

                      428- The cause of sleeping sickness:
                      a- trypanosoma gambiense
                      b- trypanosoma cruzi
                      c- trypanosoma rhodesiense
                      d- (a) & (c)
                      429- Cause Kala- azar:
                      a- leishmania tropica
                      b- leishmania braziliense
                      c- leishmania donovani
                      d- leishmania mexicana
                      430-cause oriental sore:
                      a- plasmodium ovale
                      b- leishmania tropica
                      c- leishmania donovani
                      d- trypanosoma rhodesiense
                      431- Its trophozite is shaped like a pear , has the 2 nuclei that resembles eyes
                      and 4 pairs of flagella that look like hair:
                      a- Trichomonas vaginalis
                      b- entameoba histolytica
                      c- giardia lamblia
                      d- endolimax nana
                      432- Pear shaped trophozite with 4 anterior flagella and a 5th forming the
                      outer edge of a short undulating membrane:
                      a- Trichomonas hominis
                      b- entameoba histolytica
                      c- entameoba coli
                      d- endolimax nana
                      433- sometimes it cause metastatic infection which involve liver, lung, brain
                      or other viscera:
                      a- giardia lamblia
                      b- Trichomonas vaginalis
                      c- entameoba histolytica
                      d- balantidium coli

                      434- Intestinal ciliate:
                      a- entameoba histolytica
                      b- entameoba coli
                      c- giardia lamblia
                      d- balantidium coli
                      435- Asexual multiplication of plasmodium vivax takes place in:
                      a- anopheles
                      b- sand fly
                      c- human
                      c- tse tse fly
                      436- Plasmodium falciparam is transmitted by :
                      a- triatoma megista
                      b- tse tse fly
                      c- anopheles
                      d- sand fly
                      437- Moves by pseudopods:
                      a- giardia lamblia
                      b- balantidium coli
                      c- entameoba histolytica
                      d- Trichomonas vaginalis
                      438- it's one of the round worms:
                      a- schistosoma mansoni
                      b- schistosoma haematobium
                      c- ascaris lumbricoides
                      d- fasciola hepatica
                      439- it's one of the tape worms:
                      a- ascaris lumbricoides
                      b- ancylostoma duodenal
                      c- trichuris tricura
                      d- taenia saginata

                      440- barrel shapped egg, yellow brown in color with a colorless protruding
                      mucoid plug in each end:
                      a- egg of ascaris lumbricoides
                      b- egg of ancylostoma duodenal
                      c- egg of trichuris tricura
                      d- egg of taenia saginata
                      441- large oval egg , pale yellow brown in color has a characteristic side spine
                      and contain a fully developed miracidium , the worm is:
                      a- S.mansoni
                      b- S. hematobium
                      c- A. duodenal
                      d- T. solium
                      442- large oval egg , pale yellow brown in color has an indistinct operculum
                      and contains unsegmented ovum:
                      a- S. hematobium
                      b- fasciola hepatica
                      c- heterophyes heterophyes
                      d- taenia solium
                      443- round egg , embryo is surrounded by a thick brown radially striated
                      wall , hooklets are present in the embryo:
                      a- S. hematobium
                      b- fasciola hepatica
                      c- A. duodenal
                      d- T. solium
                      444- oval colorless egg , flattened on one side and contains a larvae:
                      a- hymenelopis diminuta
                      b- dipylidium caninum
                      c- entrobius vermicularis
                      d- taenia saginata
                      445- the cause of malignant malaria:
                      a- plasmodium vivax
                      b- plasmodium ovale
                      c- plasmodium malaria
                      d- plasmodium falciparum

                      446- infection occur when infective filariform larvae penetrate the skin:
                      a- ascaris lumbricoides
                      b- ancylostoma duodenal
                      c- fasciola hepatica
                      d- heterophyes heterophyes
                      447-……..lives in the liver and bile ducts of sheep and cattle:
                      a- stronyloides stercoralis
                      b- schistosoma mansoni
                      c- fasciola hepatica
                      d- ancylostoma duodenal
                      448- segment found in stool which is white and opaque and measures 20mm
                      long by 6mm wide with uterus that has a central stem and more than 13 side
                      branches on each side…the worm is :
                      a- fasciola hepatica
                      b- trichuris trichuris
                      c- heterophyes heterophyes
                      d- taenia saginata
                      449- infection is by eating raw or under cooked fish:
                      a- fasciola hepatica
                      b- trichuris trichuris
                      c- heterophyes heterophyes
                      d- taenia solium
                      450-…………is transmitted by eating raw or under cooked beef:
                      a- heterophyes heterophyes
                      b- taenia saginata
                      c- schistosoma mansoni
                      d- ancylostoma duodenal
                      451- urine output < 400 ml/24 hours is :
                      a- polyuria
                      b- anuria
                      c- oligouria
                      d- non of the above

                      452- precipitation of urates takes place in:
                      a- acidic urine
                      b- alkaline urine
                      c- neutral urine
                      d- all of the above
                      453- it's one of the causes of persistently acidic urine:
                      a- urinary tract infection
                      b- phenylketonurea
                      c- excessive bicarbonate ingestion
                      d- excessive ingestion of soda
                      454- common cause of proteinuria:
                      a- alcoholism
                      b- fasting > 18 hours
                      c- diabetes mellitus
                      d- Bence- Jones proteins
                      455- dipstick detect acetoacetic acid and acetone which react with:
                      a- peroxides
                      b- nitroprusside
                      c- diazo compounds
                      d- indoxyl esters
                      456- in dipstick bilirubin reacts with :
                      a- nitroprusside
                      b- peroxides
                      c- indoxyl esters
                      d- diazo compounds
                      457- among the common cause of hematouria:
                      a- urogenital carcinoma
                      b- diabetes mellitus
                      c- heavy exercise
                      d- metabolic disorder

                      458- large number of hyaline cast indicated:
                      a- acute pyelonephritis
                      b- proliferative glomerulonephritis
                      c- heart failure
                      d- all of the above
                      459- red cell casts indicates:
                      a- acute pyelonephritis
                      b- proliferative glomerulonephritis
                      c- heart failure
                      d- all of the above
                      460- crystals which look like envelope :
                      a- triple phosphate
                      b- cystine
                      c- uric acid
                      d- calcium oxalate
                      461- Biuret test is done to determine:
                      a- glucose
                      b- pentose
                      c- protein
                      d- galactose
                      462- the 1st tube of synovial fluid is for:
                      a- hematology
                      b- chemistry
                      c- microbiology
                      d- microscopy
                      463- square plate like crystals with notched corners in synovial fluid indicate:
                      a- uric acid
                      b- calcium pyrophosphate
                      c- cholesterol
                      d- monosodium urate

                      464- abnormal forms in semen should not exceed:
                      a- 10 %
                      b- 5 %
                      c- 25 %
                      d- 50 %
                      465- citrate utilization test is done to assist identification of:
                      a- gram +ve bacteria
                      b- gram –ve bacteria
                      c- entrobacteria
                      d- enterococcus
                      466- the Kovac's reagent used in the following biochemical tests:
                      a- catalase
                      b- coagulase
                      c- indole
                      d- methyl red
                      467- positive results for H2S production appear as ………. Colour:
                      a- black
                      c- yellow
                      c- red
                      d- green
                      468-………. Give positive coagulase test:
                      a- streptococci
                      b- staphylococcus aureus
                      c- staphylococcus saprophyticus
                      d- Escherichia coli
                      469- methyl red test is performed with:
                      a- Erlich reagent
                      b- Kovac's reagent
                      c- Voges proskaur
                      d- non of the above

                      470-…………give positive result with urease test:
                      a- salmonella
                      b- shigella
                      c- Y. enterocolitica
                      d- all of the above
                      471-……….. test is used to differentiate between bacteroides and brucella:
                      a- indole
                      b- methyl red
                      c- H2S production
                      d- nitrate reduction
                      472- DNAase test is positive with:
                      a- streptococcus pneumonia
                      b- E.coli
                      c- staphylococcus aureus
                      d- staphylococcus epidermis
                      473-………… solution used in the gram stain technique acts as a mordant:
                      a- crystal violet
                      b- safranine
                      c- iodine
                      d- alcohol
                      474- the counter stain in Ziehl- Neelson stain is :
                      a- malachite green
                      b- methylene blue
                      c- iodine
                      d- (a) & (b)
                      475- bile solubility test is positive with :
                      a- streptococcus viridans
                      b- streptococcus pneumonia
                      c- streptococcus agalectiae
                      d- streptococcus pyrogenes

                      476-litmus milk decolorization test assist the identification of :
                      a- entrobacteria
                      b- bacteroides
                      c- brucella
                      d- enterococci
                      477-The process of getting things done through and with people operating in
                      organized group toward a common goal is the
                      a- management
                      b- Organization
                      c- Planning
                      d- None of the above
                      478- Primary objectives in the planning are directed to
                      a- the laboratory as a whole
                      b- Increase the efficiency in the performance of the lab. test
                      c- Decrease the costs in the performance of the lab. test
                      d- All of the above
                      479- Forecasting needs for staff personnel means
                      a-Prediction in relation to the kind of technician and technologist who will
                      be working in the lab.
                      b- Plan for the full utilization of efficient use of instrument
                      c- Plan for the full use of space in the lab.
                      d- None of the above
                      480-An organization
                      a- Is formed when 2 or more persons are brought together to achieve a
                      common goal
                      b- Is closely related to planning
                      c- Involves structuring activities and functions within institutions to
                      achieve the goals and objects
                      d- all of the above
                      481- The real behavior and relationships of organization members usually
                      differ from their planned behavior and relationships. It is
                      a- Formal organization
                      b- Informal organization
                      c- Space utilization

                      d- None of the above
                      482- The intra lab. System includes the following except
                      a- Calendar format
                      b- Histogram format
                      c- out of limits report sheet
                      d- Proficiency testing and computer program
                      483- The out of limits report form provides
                      a- Space for recording reagents changes
                      b- Control lot number changes
                      c- Serve as a general "dairy" of the test methodology
                      d- All of the above
                      484- Patient preparation, specimen collection and technical performance of
                      lab. test are general categories of…………..
                      a- Planning
                      b- Utilization of space
                      c-Work flow
                      d- Quality control
                      485-floor book manual includes the following except
                      a- Test name
                      b- Sample fluid
                      c-Minimum volume
                      d- Proper procedures for collecting routine and special tests
                      486- Collection procedure manual involve
                      a- Blood collection from pediatric patients
                      b- Intensive care blood collection
                      c- Isolation techniques for lab. Personnel
                      d- All of the above
                      487-On the container and \or lab requisition
                      a- Patient's full name should be put
                      b- Hospital number should be put
                      c- Date of collection should be put
                      d- All of the above

                      488 Accuracy referred to the following except
                      a- Correctness and exactness of the test
                      b- Closeness of the test to the true value
                      c- True value determined by comparison to a standard
                      d- reproducibility
                      489- Regarding precision the following is true except
                      a- Reproducibility
                      b- Closeness of the test results to one another when using the same specimen
                      c- In the clinical lab it is expressed as (SD) and coefficient of variation
                      d- The capability of the method to detect a small amount of substance with
                      some assurance
                      490- Reliability is
                      a- The ability of a method to measure only that substance being tested
                      b- The ability of the test method to maintain its accuracy despite of
                      extraneous circumstances
                      c- The ability of the method to maintain accuracy, precision and ruggedness
                      d- None of the above
                      491- - …………….. This symbol in the flow chart means
                      a- Beginning process
                      b- Decision
                      c- Manual operation
                      d- Decision mod
                      492- This symbol in the flow chart means
                      a-Beginning process
                      b- Decision
                      c- Direction flow
                      d- Document
                      493- Work load on which personnel requirements are usually based is
                      influenced by
                      a- changes in volume
                      b- Test mix
                      c- Patient population

                      d- All of the above
                      494- The physical features of the lab. one of the measures of
                      a- forecasting of personnel needs
                      b- Assessment of space utilization
                      c- Time management
                      d- None of the above
                      495- If P (E) is the probability of E we may express this definition as
                      a- P (E) = m\N
                      b- P (E) = N\m
                      c- P (E) = m X N
                      d- None of the above
                      496- When a test indicates a positive status when the true status is negative it
                      is called
                      a- positive test
                      b- False positive test
                      c- Negative test
                      d- False negative test
                      497- The specificity of a test
                      a- The probability of a positive test results or (presence of the symptoms)
                      given the presence of the disease
                      b- The probability of a negative test results or (absence of the symptoms)
                      given the absence of the disease
                      c- a and b
                      d-None of the above
                      498- The largest collection of entities for which we have an interest at a
                      particular time is called
                      a- Population
                      b- Sample
                      c- Data
                      d-All of the above
                      499- A sample is
                      a- A part of a population
                      b- The whole population
                      c- Endless population

                      d- None of the above
                      500- If we have 100 students and they are ranked by age beginning with the
                      4th student, every tenth student is chosen (the student no. 4 then 14 and 24
                      and so on) this type of sample is called
                      a- Systemically selected sample
                      b- A stratified selected sample
                      c- Simple random sample
                      d- Cluster selected sample
                      501-A point estimate is
                      a- A single numerical value used to estimate the corresponding population
                      parameter
                      b- Tow numerical values defining a range of values include the parameter
                      being estimated
                      c- a and b
                      d- None of the above
                      502-A statistical inference is
                      a- A procedure by which we reach a conclusion about population based on the
                      information obtained from the sample drawn from it
                      b- The cause behind estimation in the health science fields
                      c- Calculated data from the data of the sample that are approximation of
                      the corresponding parameter
                      d- None of the above
                      503- Estimator is
                      a- the rule that tells us how to compute the single value which is called
                      estimate
                      b- Two numerical values defining the range of values
                      c- a, b
                      d- None of the above
                      504 The table which shows the way in which values of the variables are
                      distributed among the specified class interval is called
                      a- Relative frequency
                      b- Ordered array

                      c- Frequency table
                      c- None of the above
                      505- The following are the ages of 5 patients seen in the emergency room in
                      certain day 35, 30, 55, 40, 25 years the mean of their ages is
                      a- 37 years
                      b- 30 years
                      c- 39 years
                      d- 40 years
                      506- A mathematical tool designed to facilitate complex clinical decision in
                      which many variables must be considered spontaneously is called
                      a- Reference value
                      b- Decision analysis
                      c- Quality assurance
                      d- None of the above





















                      The answers






















                      ١ A ٣٧ B
                      ٢ A ٣٨ B
                      ٣ C ٣٩ B
                      ٤ D ٤٠ D
                      ٥ C ٤١ D
                      ٦ D ٤٢ D
                      ٧ C ٤٣ C
                      ٨ C ٤٤ D
                      ٩ A ٤٥ D
                      ١٠ C ٤٦ C
                      ١١ C ٤٧ A
                      ١٢ C ٤٨ C
                      ١٣ B ٤٩ A
                      ١٤ B ٥٠ C
                      ١٥ A ٥١ B
                      ١٦ D ٥٢ C
                      ١٧ C ٥٣ A
                      ١٨ A ٥٤ D
                      ١٩ C ٥٥ B
                      ٢٠ C ٥٦ B
                      ٢١ C ٥٧ C
                      ٢٢ B ٥٨ C
                      ٢٣ C ٥٩ C
                      ٢٤ C ٦٠ A
                      ٢٥ C ٦١ D
                      ٢٦ D ٦٢ C
                      ٢٧ D ٦٣ B
                      ٢٨ B ٦٤ B
                      ٢٩ C ٦٥ D
                      ٣٠ C ٦٦ C
                      ٣١ C ٦٧ C
                      ٣٢ C ٦٨ B
                      ٣٣ D ٦٩ B
                      ٣٤ D ٧٠ C
                      ٣٥ C ٧١ A
                      ٣٦ C ٧٢ C
                      ٧٣ B ١٠٩ D
                      ٧٤ B ١١٠ B
                      ٧٥ D ١١١ D
                      ٧٦ B ١١٢ B
                      ٧٧ D ١١٣ C
                      ٧٨ B ١١٤ C
                      ٧٩ C ١١٥ A
                      ٨٠ C ١١٦ C
                      ٨١ C ١١٧ B
                      ٨٢ B ١١٨ A
                      ٨٣ C ١١٩ D
                      ٨٤ D ١٢٠ B
                      ٨٥ B ١٢١ C
                      ٨٦ B ١٢٢ B
                      ٨٧ C ١٢٣ B
                      ٨٨ C ١٢٤ D
                      ٨٩ B ١٢٥ B
                      ٩٠ D ١٢٦ C
                      ٩١ C ١٢٧ B
                      ٩٢ B ١٢٨ C
                      ٩٣ C ١٢٩ B
                      ٩٤ C ١٣٠ A
                      ٩٥ A ١٣١ D
                      ٩٦ B ١٣٢ B
                      ٩٧ B ١٣٣ B
                      ٩٨ B ١٣٤ D
                      ٩٩ C ١٣٥ A
                      ١٠٠ C ١٣٦ A
                      ١٠١ B ١٣٧ D
                      ١٠٢ C ١٣٨ B
                      ١٠٣ D ١٣٩ A
                      ١٠٤ A ١٤٠ B
                      ١٠٥ A ١٤١ D
                      ١٠٦ D ١٤٢ B
                      ١٠٧ B ١٤٣ D
                      ١٠٨ D ١٤٤ D
                      ١٤٥ A ١٨١ A
                      ١٤٦ C ١٨٢ C
                      ١٤٧ B ١٨٣ C
                      ١٤٨ C ١٨٤ C
                      ١٤٩ C ١٨٥ C
                      ١٥٠ B ١٨٦ C
                      ١٥١ B ١٨٧ C
                      ١٥٢ C ١٨٨ C
                      ١٥٣ A ١٨٩ B
                      ١٥٤ C ١٩٠ C
                      ١٥٥ D ١٩١ C
                      ١٥٦ D ١٩٢ B
                      ١٥٧ A ١٩٣ C
                      ١٥٨ C ١٩٤ D
                      ١٥٩ C ١٩٥ D
                      ١٦٠ B ١٩٦ D
                      ١٦١ D ١٩٧ C
                      ١٦٢ B ١٩٨ D
                      ١٦٣ B ١٩٩ C
                      ١٦٤ B ٢٠٠ A
                      ١٦٥ B ٢٠١ C
                      ١٦٦ C ٢٠٢ A
                      ١٦٧ B ٢٠٣ D
                      ١٦٨ A ٢٠٤ D
                      ١٦٩ A ٢٠٥ B
                      ١٧٠ C ٢٠٦ D
                      ١٧١ B ٢٠٧ D
                      ١٧٢ D ٢٠٨ D
                      ١٧٣ B ٢٠٩ D
                      ١٧٤ C ٢١٠ B
                      ١٧٥ D ٢١١ C
                      ١٧٦ B ٢١٢ D
                      ١٧٧ D ٢١٣ A
                      ١٧٨ A ٢١٤ B
                      ١٧٩ B ٢١٥ B
                      ١٨٠ C ٢١٦ B
                      ٢١٧ C ٢٥٣ C
                      ٢١٨ C ٢٥٤ B
                      ٢١٩ C ٢٥٥ A
                      ٢٢٠ D ٢٥٦ B
                      ٢٢١ A ٢٥٧ A
                      ٢٢٢ B ٢٥٨ B
                      ٢٢٣ B ٢٥٩ C
                      ٢٢٤ A ٢٦٠ C
                      ٢٢٥ A ٢٦١ B
                      ٢٢٦ C ٢٦٢ C
                      ٢٢٧ C ٢٦٣ C
                      ٢٢٨ D ٢٦٤ C
                      ٢٢٩ C ٢٦٥ B
                      ٢٣٠ D ٢٦٦ A
                      ٢٣١ B ٢٦٧ B
                      ٢٣٢ C ٢٦٨ D
                      ٢٣٣ D ٢٦٩ A
                      ٢٣٤ D ٢٧٠ D
                      ٢٣٥ C ٢٧١ C
                      ٢٣٦ D ٢٧٢ D
                      ٢٣٧ A ٢٧٣ C
                      ٢٣٨ C ٢٧٤ B
                      ٢٣٩ D ٢٧٥ B
                      ٢٤٠ B ٢٧٦ A
                      ٢٤١ C ٢٧٧ C
                      ٢٤٢ D ٢٧٨ C
                      ٢٤٣ C ٢٧٩ C
                      ٢٤٤ B ٢٨٠ B
                      ٢٤٥ B ٢٨١ D
                      ٢٤٦ C ٢٨٢ D
                      ٢٤٧ B ٢٨٣ B
                      ٢٤٨ C ٢٨٤ D
                      ٢٤٩ D ٢٨٥ D
                      ٢٥٠ A ٢٨٦ A
                      ٢٥١ C ٢٨٧ C
                      ٢٥٢ A ٢٨٨ C
                      ٢٨٩ A ٣٢٥ B
                      ٢٩٠ A ٣٢٦ B
                      ٢٩١ A ٣٢٧ C
                      ٢٩٢ C ٣٢٨ C
                      ٢٩٣ D ٣٢٩ B
                      ٢٩٤ C ٣٣٠ D
                      ٢٩٥ D ٣٣١ A
                      ٢٩٦ B ٣٣٢ C
                      ٢٩٧ C ٣٣٣ C
                      ٢٩٨ B ٣٣٤ D
                      ٢٩٩ B ٣٣٥ B
                      ٣٠٠ A ٣٣٦ B
                      ٣٠١ B ٣٣٧ B
                      ٣٠٢ D ٣٣٨ B
                      ٣٠٣ B ٣٣٩ C
                      ٣٠٤ B ٣٤٠ B
                      ٣٠٥ A ٣٤١ B
                      ٣٠٦ C ٣٤٢ A
                      ٣٠٧ B ٣٤٣ B
                      ٣٠٨ D ٣٤٤ A
                      ٣٠٩ B ٣٤٥ B
                      ٣١٠ D ٣٤٦ A
                      ٣١١ C ٣٤٧ B
                      ٣١٢ A ٣٤٨ B
                      ٣١٣ A ٣٤٩ A
                      ٣١٤ A ٣٥٠ B
                      ٣١٥ A ٣٥١ A
                      ٣١٦ B ٣٥٢ C
                      ٣١٧ B ٣٥٣ B
                      ٣١٨ B ٣٥٤ D
                      ٣١٩ B ٣٥٥ B
                      ٣٢٠ B ٣٥٦ A
                      ٣٢١ A ٣٥٧ B
                      ٣٢٢ C ٣٥٨ A
                      ٣٢٣ C ٣٥٩ D
                      ٣٢٤ C ٣٦٠ B
                      ٣٦١ C ٣٩٧ D
                      ٣٦٢ C ٣٩٨ D
                      ٣٦٣ A ٣٩٩ B
                      ٣٦٤ A ٤٠٠ B
                      ٣٦٥ B ٤٠١ C
                      ٣٦٦ B ٤٠٢ B
                      ٣٦٧ D ٤٠٣ C
                      ٣٦٨ D ٤٠٤ C
                      ٣٦٩ D ٤٠٥ B
                      ٣٧٠ D ٤٠٦ A
                      ٣٧١ C ٤٠٧ B
                      ٣٧٢ A ٤٠٨ B
                      ٣٧٣ C ٤٠٩ C
                      ٣٧٤ D ٤١٠ B
                      ٣٧٥ D ٤١١ C
                      ٣٧٦ D ٤١٢ B
                      ٣٧٧ B ٤١٣ C
                      ٣٧٨ A ٤١٤ A
                      ٣٧٩ D ٤١٥ A
                      ٣٨٠ B ٤١٦ B
                      ٣٨١ D ٤١٧ A
                      ٣٨٢ C ٤١٨ C
                      ٣٨٣ D ٤١٩ A
                      ٣٨٤ C ٤٢٠ A
                      ٣٨٥ D ٤٢١ B
                      ٣٨٦ A ٤٢٢ B
                      ٣٨٧ B ٤٢٣ A
                      ٣٨٨ C ٤٢٤ B
                      ٣٨٩ A ٤٢٥ C
                      ٣٩٠ D ٤٢٦ C
                      ٣٩١ B ٤٢٧ C
                      ٣٩٢ D ٤٢٨ D
                      ٣٩٣ C ٤٢٩ C
                      ٣٩٤ D ٤٣٠ B
                      ٣٩٥ D ٤٣١ C
                      ٣٩٦ D ٤٣٢ A
                      ٤٣٣ C ٤٦٩ C
                      ٤٣٤ D ٤٧٠ C
                      ٤٣٥ C ٤٧١ C
                      ٤٣٦ C ٤٧٢ C
                      ٤٣٧ C ٤٧٣ C
                      ٤٣٨ C ٤٧٤ D
                      ٤٣٩ D ٤٧٥ B
                      ٤٤٠ C ٤٧٦ D
                      ٤٤١ A ٤٧٧ A
                      ٤٤٢ B ٤٧٨ D
                      ٤٤٣ D ٤٧٩ A
                      ٤٤٤ C ٤٨٠ D
                      ٤٤٥ D ٤٨١ B
                      ٤٤٦ B ٤٨٢ B
                      ٤٤٧ C ٤٨٣ D
                      ٤٤٨ D ٤٨٤ D
                      ٤٤٩ C ٤٨٥ D
                      ٤٥٠ B ٤٨٦ D
                      ٤٥١ C ٤٨٧ D
                      ٤٥٢ A ٤٨٨ D
                      ٤٥٣ B ٤٨٩ D
                      ٤٥٤ D ٤٩٠ A
                      ٤٥٥ B ٤٩١ A
                      ٤٥٦ D ٤٩٢ C
                      ٤٥٧ A ٤٩٣ D
                      ٤٥٨ D ٤٩٤ B
                      ٤٥٩ B ٤٩٥ A
                      ٤٦٠ D ٤٩٦ B
                      ٤٦١ C ٤٩٧ B
                      ٤٦٢ C ٤٩٨ A
                      ٤٦٣ C ٤٩٩ A
                      ٤٦٤ C ٥٠٠ A
                      ٤٦٥ C ٥٠١ A
                      ٤٦٦ C ٥٠٢ A
                      ٤٦٧ A ٥٠٣ A
                      ٤٦٨ B ٥٠٤ C
                      ٥٠٥ A
                      ٥٠٦ B

                      تعليق


                      • #12
                        لاتنسونا من صالح الدعاء وياريت يااخ على بعد الامتحان تنزل الاسئله وتطمنا عليك ويارب ناااااااااااااااااجح بس لاتنسى بعد ماتنجح منتظرنك تطمنا

                        تعليق


                        • #13
                          المشاركة الأصلية بواسطة داليا السقاف مشاهدة المشاركة
                          لاتنسونا من صالح الدعاء وياريت يااخ على بعد الامتحان تنزل الاسئله وتطمنا عليك ويارب ناااااااااااااااااجح بس لاتنسى بعد ماتنجح منتظرنك تطمنا
                          انشاالله بس هل هاذه الاساله فقط اذاكرها

                          تعليق


                          • #14
                            اختي هل انتي اخصائيه او فنية مختبر لكي يتضح الموضوع

                            تعليق


                            • #15
                              هذه الأسئلة للأخصائيين فقط يا أخ طيب

                              تعليق

                              يعمل...
                              X